Как найти определитель матрицы любого порядка

Содержание:

  • Вычисления определителей второго порядка
  • Методы вычисления определителей третьего порядка
  • Приведение определителя к треугольному виду
  • Правило треугольника
  • Правило Саррюса
  • Разложение определителя по строке или столбцу
  • Разложение определителя по элементам строки или столбца
  • Теорема Лапласа

В общем случае правило вычисления определителей
$n$-го порядка
является довольно громоздким. Для определителей второго и третьего порядка существуют рациональные способы их вычислений.

Вычисления определителей второго порядка

Чтобы вычислить определитель матрицы второго порядка, надо от произведения
элементов главной диагонали отнять произведение
элементов побочной диагонали:

$$left| begin{array}{ll}{a_{11}} & {a_{12}} \ {a_{21}} & {a_{22}}end{array}right|=a_{11} cdot a_{22}-a_{12} cdot a_{21}$$

Пример

Задание. Вычислить определитель второго порядка
$left| begin{array}{rr}{11} & {-2} \ {7} & {5}end{array}right|$

Решение. $left| begin{array}{rr}{11} & {-2} \ {7} & {5}end{array}right|=11 cdot 5-(-2) cdot 7=55+14=69$

Ответ. $left| begin{array}{rr}{11} & {-2} \ {7} & {5}end{array}right|=69$

Методы вычисления определителей третьего порядка

Для вычисления определителей третьего порядка существует такие правила.

Правило треугольника

Схематически это правило можно изобразить следующим образом:

Определитель матрицы по правилу треугольника

Произведение элементов в первом определителе, которые соединены прямыми,
берется со знаком «плюс»; аналогично, для второго определителя — соответствующие произведения берутся со знаком «минус», т.е.

$$left| begin{array}{ccc}{a_{11}} & {a_{12}} & {a_{13}} \ {a_{21}} & {a_{22}} & {a_{23}} \ {a_{31}} & {a_{32}} & {a_{33}}end{array}right|=a_{11} a_{22} a_{33}+a_{12} a_{23} a_{31}+a_{13} a_{21} a_{32}-$$

$$-a_{11} a_{23} a_{32}-a_{12} a_{21} a_{33}-a_{13} a_{22} a_{31}$$

236

проверенных автора готовы помочь в написании работы любой сложности

Мы помогли уже 4 430 ученикам и студентам сдать работы от решения задач до дипломных на отлично! Узнай стоимость своей работы за 15 минут!

Пример

Задание. Вычислить определитель $left| begin{array}{rrr}{3} & {3} & {-1} \ {4} & {1} & {3} \ {1} & {-2} & {-2}end{array}right|$ методом треугольников.

Решение. $left| begin{array}{rrr}{3} & {3} & {-1} \ {4} & {1} & {3} \ {1} & {-2} & {-2}end{array}right|=3 cdot 1 cdot(-2)+4 cdot(-2) cdot(-1)+$

$$+3 cdot 3 cdot 1-(-1) cdot 1 cdot 1-3 cdot(-2) cdot 3-4 cdot 3 cdot(-2)=54$$

Ответ. $left| begin{array}{rrr}{3} & {3} & {-1} \ {4} & {1} & {3} \ {1} & {-2} & {-2}end{array}right|=54$

Правило Саррюса

Справа от определителя дописывают первых два столбца и произведения элементов на главной диагонали и на диагоналях, ей
параллельных, берут со знаком «плюс»; а произведения элементов побочной диагонали и диагоналей, ей параллельных,
со знаком «минус»:

$$-a_{13} a_{22} a_{31}-a_{11} a_{23} a_{32}-a_{12} a_{21} a_{33}$$

Пример

Задание. Вычислить определитель $left| begin{array}{rrr}{3} & {3} & {-1} \ {4} & {1} & {3} \ {1} & {-2} & {-2}end{array}right|$ с помощью правила Саррюса.

Решение.

$$+(-1) cdot 4 cdot(-2)-(-1) cdot 1 cdot 1-3 cdot 3 cdot(-2)-3 cdot 4 cdot(-2)=54$$

Ответ. $left| begin{array}{rrr}{3} & {3} & {-1} \ {4} & {1} & {3} \ {1} & {-2} & {-2}end{array}right|=54$

Разложение определителя по строке или столбцу

Определитель равен сумме произведений элементов строки определителя на их
алгебраические дополнения. Обычно выбирают
ту строку/столбец, в которой/ом есть нули. Строку или столбец, по которой/ому ведется разложение, будет обозначать стрелкой.

Пример

Задание. Разложив по первой строке, вычислить определитель $left| begin{array}{lll}{1} & {2} & {3} \ {4} & {5} & {6} \ {7} & {8} & {9}end{array}right|$

Решение. $left| begin{array}{lll}{1} & {2} & {3} \ {4} & {5} & {6} \ {7} & {8} & {9}end{array}right| leftarrow=a_{11} cdot A_{11}+a_{12} cdot A_{12}+a_{13} cdot A_{13}=$

$1 cdot(-1)^{1+1} cdot left| begin{array}{cc}{5} & {6} \ {8} & {9}end{array}right|+2 cdot(-1)^{1+2} cdot left| begin{array}{cc}{4} & {6} \ {7} & {9}end{array}right|+3 cdot(-1)^{1+3} cdot left| begin{array}{cc}{4} & {5} \ {7} & {8}end{array}right|=-3+12-9=0$

Ответ. $left| begin{array}{lll}{1} & {2} & {3} \ {4} & {5} & {6} \ {7} & {8} & {9}end{array}right|=0$

Этот метод позволяет вычисление определителя свести к вычислению определителя более низкого порядка.

Пример

Задание. Вычислить определитель $left| begin{array}{lll}{1} & {2} & {3} \ {4} & {5} & {6} \ {7} & {8} & {9}end{array}right|$

Решение. Выполним следующие
преобразования над строками определителя: из второй строки отнимем четыре
первых, а из третьей первую строку, умноженную на семь, в результате, согласно свойствам определителя, получим определитель,
равный данному.

$$left| begin{array}{ccc}{1} & {2} & {3} \ {4} & {5} & {6} \ {7} & {8} & {9}end{array}right|=left| begin{array}{ccc}{1} & {2} & {3} \ {4-4 cdot 1} & {5-4 cdot 2} & {6-4 cdot 3} \ {7-7 cdot 1} & {8-7 cdot 2} & {9-7 cdot 3}end{array}right|=$$

$$=left| begin{array}{rrr}{1} & {2} & {3} \ {0} & {-3} & {-6} \ {0} & {-6} & {-12}end{array}right|=left| begin{array}{ccc}{1} & {2} & {3} \ {0} & {-3} & {-6} \ {0} & {2 cdot(-3)} & {2 cdot(-6)}end{array}right|=0$$

Определитель равен нулю, так как вторая и третья строки являются пропорциональными.

Ответ. $left| begin{array}{lll}{1} & {2} & {3} \ {4} & {5} & {6} \ {7} & {8} & {9}end{array}right|=0$

Для вычисления определителей четвертого порядка и выше применяется либо разложение по строке/столбцу, либо приведение
к треугольному виду, либо с помощью теоремы Лапласа.

Разложение определителя по элементам строки или столбца

Пример

Задание. Вычислить определитель
$left| begin{array}{llll}{9} & {8} & {7} & {6} \ {5} & {4} & {3} & {2} \ {1} & {0} & {1} & {2} \ {3} & {4} & {5} & {6}end{array}right|$ , разложив его по элементам какой-то строки или какого-то столбца.

Решение. Предварительно выполним
элементарные преобразования над строками определителя, сделав
как можно больше нулей либо в строке, либо в столбце. Для этого вначале от первой строки отнимем девять третьих,
от второй — пять третьих и от четвертой — три третьих строки, получаем:

$$left| begin{array}{cccc}{9} & {8} & {7} & {6} \ {5} & {4} & {3} & {2} \ {1} & {0} & {1} & {2} \ {3} & {4} & {5} & {6}end{array}right|=left| begin{array}{cccc}{9-1} & {8-0} & {7-9} & {6-18} \ {5-5} & {4-0} & {3-5} & {2-10} \ {1} & {0} & {1} & {2} \ {0} & {4} & {2} & {0}end{array}right|=left| begin{array}{rrrr}{0} & {8} & {-2} & {-12} \ {0} & {4} & {-2} & {-8} \ {1} & {0} & {1} & {2} \ {0} & {4} & {2} & {0}end{array}right|$$

Полученный определитель разложим по элементам первого столбца:

$$left| begin{array}{rrrr}{0} & {8} & {-2} & {-12} \ {0} & {4} & {-2} & {-8} \ {1} & {0} & {1} & {2} \ {0} & {4} & {2} & {0}end{array}right|=0+0+1 cdot(-1)^{3+1} cdot left| begin{array}{rrr}{8} & {-2} & {-12} \ {4} & {-2} & {-8} \ {4} & {2} & {0}end{array}right|+0$$

Полученный определитель третьего порядка также разложим по элементам строки и столбца, предварительно получив нули,
например, в первом столбце. Для этого от первой строки отнимаем две вторые строки, а от третьей — вторую:

$$left| begin{array}{rrr}{8} & {-2} & {-12} \ {4} & {-2} & {-8} \ {4} & {2} & {0}end{array}right|=left| begin{array}{rrr}{0} & {2} & {4} \ {4} & {-2} & {-8} \ {0} & {4} & {8}end{array}right|=4 cdot(-1)^{2+2} cdot left| begin{array}{ll}{2} & {4} \ {4} & {8}end{array}right|=$$

$$=4 cdot(2 cdot 8-4 cdot 4)=0$$

Ответ. $left| begin{array}{cccc}{9} & {8} & {7} & {6} \ {5} & {4} & {3} & {2} \ {1} & {0} & {1} & {2} \ {3} & {4} & {5} & {6}end{array}right|=0$

Замечание

Последний и предпоследний определители можно было бы и не вычислять,
а сразу сделать вывод о том, что они равны нулю, так как содержат пропорциональные строки.

Приведение определителя к треугольному виду

С помощью элементарных преобразований над строками или столбцами определитель приводится к треугольному виду и тогда его
значение, согласно свойствам определителя, равно произведению
элементов стоящих на главной диагонали.

Пример

Задание. Вычислить определитель
$Delta=left| begin{array}{rrrr}{-2} & {1} & {3} & {2} \ {3} & {0} & {-1} & {2} \ {-5} & {2} & {3} & {0} \ {4} & {-1} & {2} & {-3}end{array}right|$ приведением его к треугольному виду.

Решение. Сначала делаем нули в первом столбце под главной диагональю. Все преобразования
будет выполнять проще, если элемент $a_{11}$ будет
равен 1. Для этого мы поменяем местами первый и второй столбцы определителя, что, согласно свойствам определителя,
приведет к тому, что он сменит знак на противоположный:

$$Delta=left| begin{array}{rrrr}{-2} & {1} & {3} & {2} \ {3} & {0} & {-1} & {2} \ {-5} & {2} & {3} & {0} \ {4} & {-1} & {2} & {-3}end{array}right|=-left| begin{array}{rrrr}{1} & {-2} & {3} & {2} \ {0} & {3} & {-1} & {2} \ {2} & {-5} & {3} & {0} \ {-1} & {4} & {2} & {-3}end{array}right|$$

Далее получим нули в первом столбце, кроме элемента $a_{11}$ ,
для этого из третьей строки вычтем две первых, а к четвертой строке прибавим первую, будем иметь:

$$Delta=-left| begin{array}{rrrr}{1} & {-2} & {3} & {2} \ {0} & {3} & {-1} & {2} \ {0} & {-1} & {-3} & {-4} \ {0} & {2} & {5} & {-1}end{array}right|$$

Далее получаем нули во втором столбце на месте элементов, стоящих под главной диагональю. И снова, если
диагональный элемент будет равен $pm 1$ , то
вычисления будут более простыми. Для этого меняем местами вторую и третью строки (и при этом меняется на
противоположный знак определителя):

$$Delta=left| begin{array}{rrrr}{1} & {-2} & {3} & {2} \ {0} & {-1} & {-3} & {-4} \ {0} & {3} & {-1} & {2} \ {0} & {2} & {5} & {-1}end{array}right|$$

Далее делаем нули во втором столбце под главной диагональю, для этого поступаем следующим образом:
к третьей строке прибавляем три вторых, а к четвертой — две вторых строки, получаем:

$$Delta=left| begin{array}{rrrr}{1} & {-2} & {3} & {2} \ {0} & {-1} & {-3} & {-4} \ {0} & {0} & {-10} & {-10} \ {0} & {0} & {-1} & {-9}end{array}right|$$

Далее из третьей строки выносим (-10) за определитель и делаем нули в третьем столбце под
главной диагональю, а для этого к последней строке прибавляем третью:

$$Delta=-10 left| begin{array}{rrrr}{1} & {-2} & {3} & {2} \ {0} & {-1} & {-3} & {-4} \ {0} & {0} & {1} & {1} \ {0} & {0} & {-1} & {-9}end{array}right|=$$

$$=-10 cdot left| begin{array}{cccc}{1} & {-2} & {3} & {2} \ {0} & {-1} & {-3} & {-4} \ {0} & {0} & {1} & {1} \ {0} & {0} & {0} & {-8}end{array}right|=(-10) cdot 1 cdot(-1) cdot 1 cdot(-8)=-80$$

Ответ. $Delta=-80$

Теорема Лапласа

Теорема

Пусть $Delta$ — определитель
$n$-го порядка. Выберем в нем произвольные
$k$ строк (или столбцов), причем
$k leq n-1$ . Тогда сумма произведений всех
миноров
$k$-го порядка, которые содержатся в выбранных
$k$ строках (столбцах), на их
алгебраические дополнения равна определителю.

Пример

Задание. Используя теорему Лапласа, вычислить определитель
$left| begin{array}{rrrrr}{2} & {3} & {0} & {4} & {5} \ {0} & {1} & {0} & {-1} & {2} \ {3} & {2} & {1} & {0} & {1} \ {0} & {4} & {0} & {-5} & {0} \ {1} & {1} & {2} & {-2} & {1}end{array}right|$

Решение. Выберем в данном определителе пятого порядка две строки —
вторую и третью, тогда получаем (слагаемые, которые равны нулю, опускаем):

$$left| begin{array}{rrrrr}{2} & {3} & {0} & {4} & {5} \ {0} & {1} & {0} & {-1} & {2} \ {3} & {2} & {1} & {0} & {1} \ {0} & {4} & {0} & {-5} & {0} \ {1} & {1} & {2} & {-2} & {1}end{array}right|=left| begin{array}{cc}{1} & {-1} \ {4} & {-5}end{array}right| cdot(-1)^{2+4+2+4} cdot left| begin{array}{ccc}{2} & {0} & {5} \ {3} & {1} & {1} \ {1} & {2} & {1}end{array}right|+$$

$$+left| begin{array}{ll}{1} & {2} \ {4} & {0}end{array}right| cdot(-1)^{2+4+2+5} cdot left| begin{array}{rrr}{2} & {0} & {4} \ {3} & {1} & {0} \ {1} & {2} & {-2}end{array}right|+left| begin{array}{cc}{-1} & {2} \ {-5} & {0}end{array}right| cdot(-1)^{2+4+5} cdot left| begin{array}{ccc}{2} & {3} & {0} \ {3} & {2} & {1} \ {1} & {1} & {2}end{array}right|=$$

$$=-23+128+90=195$$

Ответ. $left| begin{array}{rrrrr}{2} & {3} & {0} & {4} & {5} \ {0} & {1} & {0} & {-1} & {2} \ {3} & {2} & {1} & {0} & {1} \ {0} & {4} & {0} & {-5} & {0} \ {1} & {1} & {2} & {-2} & {1}end{array}right|=195$

Читать дальше: обратная матрица.

Содержание:

Определители II и III порядка

Определение: Определителем порядка n называется число (выражение), записанное в виде квадратной таблицы, имеющей n строк и n столбцов, которая раскрывается по определенному правилу.

Определитель матрицы - определение и вычисление с примерами решения

Числа Определитель матрицы - определение и вычисление с примерами решения

Определение: Определителем II порядка называется число (выражение), записанное в виде квадратной таблицы размером 2×2, т.е. имеющая 2 строки и 2 столбца.

Определение: Определитель II порядка вычисляется по правилу: из произведения элементов, стоящих на главной диагонали, надо вычесть произведение элементов, стоящих на побочной диагонали: Определитель матрицы - определение и вычисление с примерами решения

Пример:

Определитель матрицы - определение и вычисление с примерами решения

Определение: Определителем III порядка называется число (выражение), записанное в виде квадратной таблицы размером 3×3, то есть имеющей 3 строки и 3 столбца.

Определитель III порядка вычисляется по правилу Саррюса: за определителем выписывают первый и второй столбцы, затем из суммы произведений элементов, стоящих на главной диагонали ей параллельных диагоналях, надо вычесть сумму произведений элементов, стоящих на побочной диагонали и ей параллельных: Определитель матрицы - определение и вычисление с примерами решения

Определитель матрицы - определение и вычисление с примерами решения

Пример:

Определитель матрицы - определение и вычисление с примерами решения

Определение: Минором Определитель матрицы - определение и вычисление с примерами решения элемента Определитель матрицы - определение и вычисление с примерами решения называется определитель порядка (n-1), который получается из исходного определителя порядка n путем вычеркивания строки i и столбца j, на пересечении которых стоит элемент Определитель матрицы - определение и вычисление с примерами решения

Определитель матрицы - определение и вычисление с примерами решения

Пример:

Найти миноры элементов Определитель матрицы - определение и вычисление с примерами решенияи Определитель матрицы - определение и вычисление с примерами решенияопределителя из Примера 2. Вычеркивая в определителе строку 1 и столбец 2:Определитель матрицы - определение и вычисление с примерами решения получим минорОпределитель матрицы - определение и вычисление с примерами решения Поступая аналогично со строкой 3 и столбцом 3, получим минор Определитель матрицы - определение и вычисление с примерами решения

Пример:

Найти миноры элементов Определитель матрицы - определение и вычисление с примерами решения и Определитель матрицы - определение и вычисление с примерами решения определителя Определитель матрицы - определение и вычисление с примерами решения Исходя из определения минора Определитель матрицы - определение и вычисление с примерами решения получаем Определитель матрицы - определение и вычисление с примерами решения аналогично найдем минор Определитель матрицы - определение и вычисление с примерами решения

Определение: Алгебраическим дополнением Определитель матрицы - определение и вычисление с примерами решенияэлемента Определитель матрицы - определение и вычисление с примерами решения называется произведение минора этого элемента на Определитель матрицы - определение и вычисление с примерами решения т.е. Определитель матрицы - определение и вычисление с примерами решения

Замечание: Из определения алгебраического дополнения следует, что алгебраическое дополнение совпадает со своим минором, если сумма Определитель матрицы - определение и вычисление с примерами решения является четным числом, и противоположно ему по знаку, если сумма Определитель матрицы - определение и вычисление с примерами решения — нечетное число.

Определение: Транспонированным определителем n-го порядка называется определитель порядка n, полученный из исходного определителя путем замены строк на соответствующие столбцы, а столбцов на соответствующие строки.

Если Определитель матрицы - определение и вычисление с примерами решения

Пример:

Найти определитель, транспонированный к определителюОпределитель матрицы - определение и вычисление с примерами решения Из определения транспонированного определителя Определитель матрицы - определение и вычисление с примерами решения

Свойства определителей

1. Величина транспонированного определителя равна величине исходного определителя. Пусть Определитель матрицы - определение и вычисление с примерами решения Отсюда видно, что Определитель матрицы - определение и вычисление с примерами решения

2. Перестановка местами двух строк (столбцов) изменяет знак определителя на противоположный. Пусть Определитель матрицы - определение и вычисление с примерами решения

Определитель матрицы - определение и вычисление с примерами решения

Если поменять местами строки (столбцы) четное число раз, то величина и знак определителя не меняется. Нечетная перестановка местами строк (столбцов) не меняет величину определителя, но изменяет его знак на противоположный.

3. Определитель, содержащий две (или более) одинаковых строки (столбца), равен нулю. Если определитель содержит два одинаковых столбца, то Определитель матрицы - определение и вычисление с примерами решения Определитель матрицы - определение и вычисление с примерами решения

4. Для того чтобы умножить определитель на число k, достаточно умножить на это число все элементы какой-либо одной строки (столбца). Обратно: если все элементы какой-либо строки (столбца) имеют общий множитель k, то его можно вынести за знак определителя.

Докажем это свойство: Определитель матрицы - определение и вычисление с примерами решения

5. Если две каких-либо строки (столбца) пропорциональны, то определитель равен нулю.

Пусть в определителе II порядка первая и вторая строки пропорциональны, тогда Определитель матрицы - определение и вычисление с примерами решения

6. Если все элементы какой-либо строки (столбца) равны нулю, то определитель равен нулю.

Пусть в определителе II порядка все элементы первой строки равны нулю, тогда Определитель матрицы - определение и вычисление с примерами решения

7. Если элементы какой-либо строки (или столбца) можно представить в виде двух слагаемых, то сам определитель можно представить в виде суммы двух определителей. Если Определитель матрицы - определение и вычисление с примерами решения Доказать самостоятельно.

8. Если все элементы какой-либо строки (столбца) умножить на вещественное число к и прибавить k соответствующим элементам другой строки (соответственно, столбца), то величина определителя не изменится.

Умножим элементы второго столбца на вещественное число k и прибавим результат умножения к соответствующим элементам первого столбца, получимОпределитель матрицы - определение и вычисление с примерами решения

Второй определитель равен нулю по свойству 5.

Замечание: Данное свойство применяется для обнуления всех элементов какой-либо строки (столбца) за исключением одного (метод обнуления), что существенно снижает трудоемкость вычисления определителей порядка выше 3 (см. также свойство 9.).

9. [Метод раскрытия определителя по элементам какой-либо строки (или столбца); универсальный способ вычисления определителя любого порядка]. Определитель любого порядка равен сумме произведений элементов какой-либо строки (столбца) на их алгебраические дополнения:

Определитель матрицы - определение и вычисление с примерами решения

Пример:

Вычислить определитель Определитель матрицы - определение и вычисление с примерами решения по элементам 3 строки и по элементам 2 столбца.

Решение:

Воспользуемся свойством 9.: раскроем определитель по элементам 3 строки Определитель матрицы - определение и вычисление с примерами решения Вычислим определитель по элементам 2 столбцаОпределитель матрицы - определение и вычисление с примерами решения

Определитель матрицы - определение и вычисление с примерами решения

Из полученных результатов видно, что свойство 9. является универсальным методом вычисления любых определителей по элементам любой строки или столбца.

Используя свойство 8. можно обнулить все элементы какой-либо строки (столбца) за исключением одного (метод обнуления), а затем раскрыть определитель по элементам этой строки, воспользовавшись свойством 9.

Пример:

Вычислить определитель Определитель матрицы - определение и вычисление с примерами решения

Решение:

Обнулим элементы в третьей строке, для чего выполним следующие действия: Определитель матрицы - определение и вычисление с примерами решения (по свойству 4. из третьей строки вынесем множитель 2) Определитель матрицы - определение и вычисление с примерами решенияиспользуя свойство 8., умножим все элементы второго столбца на 1.5 и прибавим к соответствующим элементам третьего столбца, получим) Определитель матрицы - определение и вычисление с примерами решения

(по свойству 4. из третьего столбца вынесем множитель 0,5, тогда множитель перед определителем станет равным 1) Определитель матрицы - определение и вычисление с примерами решения

(раскроем определитель по элементам третьей строки: Определитель матрицы - определение и вычисление с примерами решениявыше из определителя третьего порядка вычеркнута третья строка с нулями и второй столбец, т.е. показан необходимый для дальнейших вычислений минор Определитель матрицы - определение и вычисление с примерами решения Таким образом, метод обнуления позволяет значительно ускорить процесс вычисления любого определителя.

Пример:

Решить уравнение Определитель матрицы - определение и вычисление с примерами решения

Решение:

Вычислим определители второго и третьего порядков согласно вышеописанным правилам:

Определитель матрицы - определение и вычисление с примерами решения

Найденные величины подставим в исходное уравнение

Определитель матрицы - определение и вычисление с примерами решения

Пример:

Решить неравенство Определитель матрицы - определение и вычисление с примерами решения

Решение:

Вычислим определители второго и третьего порядков согласно вышеописанным правилам:Определитель матрицы - определение и вычисление с примерами решения

Определитель матрицы - определение и вычисление с примерами решения

Найденные величины подставим в исходное неравенство Определитель матрицы - определение и вычисление с примерами решения

Пример:

Вычислить определитель четвертого порядка (аналогично выполнить такие же действия с определителем третьего порядка), преобразовав его так, чтобы три элемента некоторого ряда равнялись нулю, и вычислить полученный определитель по элементам этого ряда: Определитель матрицы - определение и вычисление с примерами решения

Решение:

Во второй строке исходного определителя присутствуют 1 и 0, поэтому обнуление элементов будем производить в этой строке (при обнулении элементов в строке действия производят со столбцами и наоборот): Определитель матрицы - определение и вычисление с примерами решения — строка обнуления; Определитель матрицы - определение и вычисление с примерами решения— столбцы, с которыми производят действия)=Определитель матрицы - определение и вычисление с примерами решения

(по методу обнуления раскроем определитель по элементам 2-ой строки (Определитель матрицы - определение и вычисление с примерами решения — цифры, с которыми производятся действия))Определитель матрицы - определение и вычисление с примерами решения

Определитель матрицы - определение и вычисление с примерами решения

Определитель матрицы - определение и вычисление с примерами решения (по универсальному методу раскроем определитель по элементам третьей строки)Определитель матрицы - определение и вычисление с примерами решения

Определитель матрицы - определение и вычисление с примерами решения

Определители

Перестановкой чисел 1, 2,…, n называется любое расположение этих чисел в определенном порядке. В элементарной алгебре доказывается, что число всех перестановок, которые можно образовать из n чисел, равно 12…n = n!. Например, из трех чисел 1, 2, 3 можно образовать 3!=6 перестановок: 123, 132, 312, 321, 231, 213. Говорят, что в данной перестановке числа i и j составляют инверсию (беспорядок), если i>j, но i стоит в этой перестановке раньше j, то есть если большее число стоит левее меньшего.

Перестановка называется четной (или нечетной), если в ней соответственно четно (нечетно) общее число инверсий. Операция, посредством которой от одной перестановки переходят к другой, составленной из тех же n чисел, называется подстановкой n-ой степени.

Подстановка, переводящая одну перестановку в другую, записывается двумя строками в общих скобках, причем числа, занимающие одинаковые места в рассматриваемых перестановках, называются соответствующими и пишутся одно под другим. Например, символ Определитель матрицы - определение и вычисление с примерами решения обозначает подстановку в которой 3 переходит в Определитель матрицы - определение и вычисление с примерами решения

Подстановка называется четной (или нечетной), если общее число инверсий в обеих строках подстановки четно (нечетно). Всякая подстановка n-ой степени может быть записана в виде Определитель матрицы - определение и вычисление с примерами решения т.е. с натуральным расположением чисел в верхней строке.

Пусть нам дана квадратная матрица порядка n Определитель матрицы - определение и вычисление с примерами решения

Рассмотрим все возможные произведения по n элементов этой матрицы, взятых по одному и только по одному из каждой строки и каждого столбца, т.е. произведений вида: Определитель матрицы - определение и вычисление с примерами решения

где индексы Определитель матрицы - определение и вычисление с примерами решения составляют некоторую перестановку из чисел 1, 2,…,n. Число таких произведений равно числу различных перестановок из n символов, т.е. равно n!. Знак произведения (4.4) равен (-1)q где q — число инверсий в перестановке вторых индексов элементов.

Определителем n-го порядка, соответствующим матрице (4.3), называется алгебраическая сумма n! членов вида (4.4). Для записи определителя употребляется символ Определитель матрицы - определение и вычисление с примерами решения (детерминант, или определитель, матрицы А).

Свойства определителей:

  1. Определитель не меняется при транспонировании.
  2. Если одна из строк определителя состоит из нулей, то определитель равен нулю.
  3. Если в определителе переставить две строки, определитель поменяет знак.
  4. Определитель, содержащий две одинаковые строки, равен нулю.
  5. Если все элементы некоторой строки определителя умножить на некоторое число Определитель матрицы - определение и вычисление с примерами решения то сам определитель умножится на Определитель матрицы - определение и вычисление с примерами решения
  6. Определитель, содержащий две пропорциональные строки, равен нулю.
  7. Если все элементы i-й строки определителя представлены в виде суммы двух слагаемых Определитель матрицы - определение и вычисление с примерами решения то определитель равен сумме определителей, у которых все строки, кроме i-ой, — такие же, как в заданном определителе, а i-я строка в одном из слагаемых состоит из элементов Определитель матрицы - определение и вычисление с примерами решения в другом — из элементов Определитель матрицы - определение и вычисление с примерами решения
  8. Определитель не меняется, если к элементам одной из его строк прибавляются соответствующие элементы другой строки, умноженные на одно и то же число.

Замечание. Все свойства остаются справедливыми, если вместо строк взять столбцы.

Минором Определитель матрицы - определение и вычисление с примерами решения элемента Определитель матрицы - определение и вычисление с примерами решения определителя d n-го порядка называется определитель порядка n-1, который получается из d вычеркиванием строки и столбца, содержащих данный элемент.

Алгебраическим дополнением элемента Определитель матрицы - определение и вычисление с примерами решения определителя d называется его минор Определитель матрицы - определение и вычисление с примерами решения взятый со знаком Определитель матрицы - определение и вычисление с примерами решения Алгебраическое дополнение элемента Определитель матрицы - определение и вычисление с примерами решения будем обозначать Определитель матрицы - определение и вычисление с примерами решения Таким образом, Определитель матрицы - определение и вычисление с примерами решения

Способы практического вычисления определителей, основанные на том, что определитель порядка n может быть выражен через определители более низких порядков, дает следующая теорема.

  • Заказать решение задач по высшей математике

Теорема (разложение определителя по строке или столбцу).

Определитель равен сумме произведений всех элементов произвольной его строки (или столбца) на их алгебраические дополнения. Иначе говоря, имеет место разложение d по элементам i-й строки Определитель матрицы - определение и вычисление с примерами решения или j- го столбца Определитель матрицы - определение и вычисление с примерами решения

В частности, если все элементы строки (или столбца), кроме одного, равны нулю, то определитель равен этому элементу, умноженному на его алгебраическое дополнение.

Пример:

Не вычисляя определителя Определитель матрицы - определение и вычисление с примерами решения показать, что он равен нулю.

Решение:

Вычтем из второй строки первую, получим определитель Определитель матрицы - определение и вычисление с примерами решения равный исходному. Если из третьей строки также вычесть первую, то получится определитель Определитель матрицы - определение и вычисление с примерами решения в котором две строки пропорциональны.

Такой определитель равен нулю.

Пример:

Вычислить определитель Определитель матрицы - определение и вычисление с примерами решения разложив его по элементам второго столбца.

Решение:

Разложим определитель по элементам второго столбца: Определитель матрицы - определение и вычисление с примерами решения

Пример:

Вычислить определитель Определитель матрицы - определение и вычисление с примерами решения в котором все элементы по одну сторону от главной диагонали равны нулю.

Решение:

Разложим определитель А по первой строке:

Определитель матрицы - определение и вычисление с примерами решения

Определитель, стоящий справа, можно снова разложить по первой строке, тогда получим: Определитель матрицы - определение и вычисление с примерами решения

И так далее. После n шагов придем к равенству Определитель матрицы - определение и вычисление с примерами решения

Пример:

Вычислить определитель Определитель матрицы - определение и вычисление с примерами решения

Решение:

Если к каждой строке определителя, начиная со второй, прибавить первую строку, то получится определитель, в котором все элементы, находящиеся ниже главной диагонали, будут равны нулю. А именно, получим определитель: Определитель матрицы - определение и вычисление с примерами решения равный исходному.

Рассуждая, как в предыдущем примере найдем, что он равен произведению элементов главной диагонали, т.е. n!. Способ, с помощью которого вычислен данный определитель, называется способом приведения к треугольному виду.

——- в вышмате

Определители. Алгебраические дополнения

Внимание! Понятие определителя вводится только для квадратной матрицы.

Матрица называется квадратной порядка n, если количество ее строк совпадает с количеством столбцов и равно n.

Элементы квадратной матрицы, имеющие одинаковые значения индексов, составляют главную диагональ. Элементы квадратной матрицы порядка n, сумма индексов каждого из которых равна n+1, составляют побочную диагональ.

Определитель матрицы Определитель матрицы - определение и вычисление с примерами решения обозначается одним из следующих символов: Определитель матрицы - определение и вычисление с примерами решения

Внимание! Определитель — это число, характеризующее квадратную мат- рицу.

Определитель матрицы второго порядка равен разности элементов главной и побочной диагоналей соответственно:

Определитель матрицы - определение и вычисление с примерами решения

Определитель матрицы третьего порядка равен сумме элементов главной диагонали и элементов, расположенных в вершинах треугольников с основаниями, параллельными главной диагонали, а также разности элементов побочной диагонали и элементов, расположенных в вершинах треугольников с основаниями, параллельными побочной диагонали. Определитель матрицы - определение и вычисление с примерами решения

Схематично это правило изображается так (правило треугольника): Определитель матрицы - определение и вычисление с примерами решения

Например,

Определитель матрицы - определение и вычисление с примерами решения Квадратная матрица называется верхней (нижней) треугольной, если все элементы, стоящие под (над) главной диагональю равны нулю.

Отметим некоторые свойства определителя.

  1. Определитель треугольной матрицы равен произведению элементов главной диагонали.
  2. При транспонировании матрицы ее определитель не изменяется.
  3. От перестановки двух рядов (строк или столбцов) определитель меняет знак.
  4. Общий множитель всех элементов некоторого ряда определителя можно выносить за знак определителя.
  5. Если все элементы какого-нибудь ряда матрицы равны нулю, то определитель равен нулю.
  6. Определитель, содержащий два пропорциональных ряда, равен нулю.
  7. Определитель не изменится, если к элементам какого-либо ряда прибавить соответствующие элементы другого ряда, умноженные на одно и то же число.
  8. Определитель произведения двух матриц одинакового порядка равен произведению определителей этих матриц.

Минором элемента Определитель матрицы - определение и вычисление с примерами решения определителя n-го порядка называется определитель (n-l)-ro порядка, получаемый вычеркиванием i-й строки и j-ro столбца, на пересечении которых стоит этот элемент. Обозначение: Определитель матрицы - определение и вычисление с примерами решения

Алгебраическим дополнением элемента Определитель матрицы - определение и вычисление с примерами решения называется его минор, умноженный на Определитель матрицы - определение и вычисление с примерами решения Обозначение: Определитель матрицы - определение и вычисление с примерами решения

Определитель матрицы - определение и вычисление с примерами решения

Теорема разложения.

Определитель матрицы равен сумме произведений элементов любого ряда на их алгебраические дополнения.

Пример №2

Вычислить определитель, разлагая его по элементам первой строки: Определитель матрицы - определение и вычисление с примерами решения

Решение:

По теореме разложения Определитель матрицы - определение и вычисление с примерами решения

Найдем алгебраические дополнения элементов матрицы А: Определитель матрицы - определение и вычисление с примерами решения

Следовательно,

Определитель матрицы - определение и вычисление с примерами решения

Для вычисления определителя порядка выше третьего удобно пользоваться теоремой разложения (метод понижения порядка) или методом приведения определителя к треугольному виду.

Пример №3

Вычислить определитель, приведя его к треугольному виду:

Определитель матрицы - определение и вычисление с примерами решения

Решение:

Применяя свойство 6 определителей, преобразуем последовательно второй, третий, четвертый столбцы матрицы. Определитель матрицы - определение и вычисление с примерами решения

  1. прибавили ко второму столбцу первый, умноженный на -2;
  2. прибавили к третьему столбцу первый, умноженный на -3;
  3. прибавили к четвертому столбцу первый, умноженный на -4;
  4. применили свойство 1 определителей.
  • Критерий совместности Кронекера-Капелли
  • Формулы Крамера
  • Матричный метод
  • Экстремум функции
  • Пределы в математике
  • Функции многих переменных
  • Уравнения прямых и кривых на плоскости
  • Плоскость и прямая в пространстве

Определитель матрицы и его свойства

8 февраля 2018

В этом уроке мы детально рассмотрим несколько ключевые вопросов и определений, благодаря чему вы раз и навсегда разберётесь и с матрицами, и с определителями, и со всеми их свойствами.

Определители — центральное понятие в алгебре матриц. Подобно формулам сокращённого умножения, они будут преследовать вас на протяжении всего курса высшей математики. Поэтому читаем, смотрим и разбираемся досконально.:)

И начнём мы с самого сокровенного — а что такое матрица? И как правильно с ней работать.

Правильная расстановка индексов в матрице

Матрица — это просто таблица, заполненная числами. Нео тут ни при чём.

Одна из ключевых характеристик матрицы — это её размерность, т.е. количество строк и столбцов, из которых она состоит. Обычно говорят, что некая матрица $A$ имеет размер $left[ mtimes n right]$, если в ней имеется $m$ строк и $n$ столбцов. Записывают это так:

[A=left[ mtimes n right]]

Или вот так:

[A=left( {{a}_{ij}} right),quad 1le ile m;quad 1le jle n.]

Бывают и другие обозначения — тут всё зависит от предпочтений лектора/ семинариста/ автора учебника. Но в любом случае со всеми этими $left[ mtimes n right]$ и ${{a}_{ij}}$ возникает одна и та же проблема:

Какой индекс за что отвечает? Сначала идёт номер строки, затем — столбца? Или наоборот?

При чтении лекций и учебников ответ будет казаться очевидным. Но когда на экзамене перед вами — только листик с задачей, можно переволноваться и внезапно запутаться.

Поэтому давайте разберёмся с этим вопросом раз и навсегда. Для начала вспомним обычную систему координат из школьного курса математики:

Введение системы координат на плоскости

Помните её? У неё есть начало координат (точка $O=left( 0;0 right)$) оси $x$и $y$, а каждая точка на плоскости однозначно определяется по координатам: $A=left( 1;2 right)$, $B=left( 3;1 right)$ и т.д.

А теперь давайте возьмём эту конструкцию и поставим её рядом с матрицей так, чтобы начало координат находилось в левом верхнем углу. Почему именно там? Да потому что открывая книгу, мы начинаем читать именно с левого верхнего угла страницы — запомнить это легче лёгкого.

Но куда направить оси? Мы направим их так, чтобы вся наша виртуальная «страница» была охвачена этими осями. Правда, для этого придётся повернуть нашу систему координат. Единственно возможный вариант такого расположения:

Наложение системы координат на матрицу

Теперь всякая клетка матрицы имеет однозначные координаты $x$ и $y$. Например запись ${{a}_{24}}$ означает, что мы обращаемся к элементу с координатами $x=2$ и $y=4$. Размеры матрицы тоже однозначно задаются парой чисел:

Определение индексов в матрице

Просто всмотритесь в эту картинку внимательно. Поиграйтесь с координатами (особенно когда будете работать с настоящими матрицами и определителями) — и очень скоро поймёте, что даже в самых сложных теоремах и определениях вы прекрасно понимаете, о чём идёт речь.

Разобрались? Что ж, переходим к первому шагу просветления — геометрическому определению определителя.:)

Геометрическое определение

Прежде всего хотел бы отметить, что определитель существует только для квадратных матриц вида $left[ ntimes n right]$. Определитель — это число, которое cчитается по определённым правилам и является одной из характеристик этой матрицы (есть другие характеристики: ранг, собственные вектора, но об этом в других уроках).

Ну и что это за характеристика? Что он означает? Всё просто:

Определитель квадратной матрицы $A=left[ ntimes n right]$ — это объём $n$-мерного параллелепипеда, который образуется, если рассмотреть строки матрицы в качестве векторов, образующих рёбра этого параллелепипеда.

Например, определитель матрицы размера 2×2 — это просто площадь параллелограмма, а для матрицы 3×3 это уже объём 3-мерного параллелепипеда — того самого, который так бесит всех старшеклассников на уроках стереометрии.

На первый взгляд это определение может показаться совершенно неадекватным. Но давайте не будем спешить с выводами — глянем на примеры. На самом деле всё элементарно, Ватсон:

Задача. Найдите определители матриц:

[left| begin{matrix} 1 & 0 \ 0 & 3 \end{matrix} right|quad left| begin{matrix} 1 & -1 \ 2 & 2 \end{matrix} right|quad left| begin{matrix}2 & 0 & 0 \ 1 & 3 & 0 \ 1 & 1 & 4 \end{matrix} right|]

Решение. Первые два определителя имеют размер 2×2. Значит, это просто площади параллелограммов. Начертим их и посчитаем площадь.

Первый параллелограмм построен на векторах ${{v}_{1}}=left( 1;0 right)$ и ${{v}_{2}}=left( 0;3 right)$:

Определитель 2×2 — это площадь параллелограмма

Очевидно, это не просто параллелограмм, а вполне себе прямоугольник. Его площадь равна

[S=1cdot 3=3]

Второй параллелограмм построен на векторах ${{v}_{1}}=left( 1;-1 right)$ и ${{v}_{2}}=left( 2;2 right)$. Ну и что с того? Это тоже прямоугольник:

Ещё один определитель 2×2

Стороны этого прямоугольника (по сути — длины векторов) легко считаются по теореме Пифагора:

[begin{align} & left| {{v}_{1}} right|=sqrt{{{1}^{2}}+{{left( -1 right)}^{2}}}=sqrt{2}; \ & left| {{v}_{2}} right|=sqrt{{{2}^{2}}+{{2}^{2}}}=sqrt{8}=2sqrt{2}; \ & S=left| {{v}_{1}} right|cdot left| {{v}_{2}} right|=sqrt{2}cdot 2sqrt{2}=4. \end{align}]

Осталось разобраться с последним определителем — там уже матрица 3×3. Придётся вспоминать стереометрию:

Определитель 3×3 — это объём параллелепипеда

Выглядит мозговыносяще, но по факту достаточно вспомнить формулу объёма параллелепипеда:

[V=Scdot h]

где $S$ — площадь основания (в нашем случае это площадь параллелограмма на плоскости $OXY$), $h$ — высота, проведённая к этому основанию (по сути, $z$-координата вектора ${{v}_{3}}$).

Площадь параллелограмма (мы начертили его отдельно) тоже считается легко:

[begin{align} & S=2cdot 3=6; \ & V=Scdot h=6cdot 4=24. \end{align}]

Вот и всё! Записываем ответы.

Ответ: 3; 4; 24.

Небольшое замечание по поводу системы обозначений. Кому-то наверняка не понравится, что я игнорирую «стрелочки» над векторами. Якобы так можно спутать вектор с точкой или ещё с чем.

Но давайте серьёзно: мы с вами уже взрослые мальчики и девочки, поэтому из контекста прекрасно понимаем, когда речь идёт о векторе, а когда — о точке. Стрелки лишь засоряют повествование, и без того под завязку напичканное математическими формулами.

И ещё. В принципе, ничто не мешает рассмотреть и определитель матрицы 1×1 — такая матрица представляет собой просто одну клетку, а число, записанное в этой клетке, и будет определителем. Но тут есть важное замечание:

В отличие от классического объёма, определитель даст нам так называемый «ориентированный объём», т.е. объём с учётом последовательности рассмотрения векторов-строк.

И если вы хотите получить объём в классическом смысле этого слова, придётся взять модуль определителя, но сейчас не стоит париться об этом — всё равно через несколько секунд мы научимся считать любой определитель с любыми знаками, размерами и т.д.:)

Алгебраическое определение

При всей красоте и наглядности геометрического подхода у него есть серьёзный недостаток: он ничего не говорит нам о том, как этот самый определитель считать.

Поэтому сейчас мы разберём альтернативное определение — алгебраическое. Для этого нам потребуется краткая теоретическая подготовка, зато на выходе мы получим инструмент, позволяющий считать в матрицах что и как угодно.

Правда, там появится новая проблема… но обо всём по порядку.

Перестановки и инверсии

Давайте выпишем в строчку числа от 1 до $n$. Получится что-то типа этого:

[1;2;3;4;5;…;n-1;n]

Теперь (чисто по приколу) поменяем парочку чисел местами. Можно поменять соседние:

[1;3;2;4;5;…;n-1;n]

А можно — не особо соседние:

[n;2;3;4;5;…;n-1;1]

И знаете, что? А ничего! В алгебре эта хрень называется перестановкой. И у неё есть куча свойств.

Определение. Перестановка длины $n$ — строка из $n$ различных чисел, записанных в любой последовательности. Обычно рассматриваются первые $n$ натуральных чисел (т.е. как раз числа 1, 2, …, $n$), а затем их перемешивают для получения нужной перестановки.

Обозначаются перестановки так же, как и векторы — просто буквой и последовательным перечислением своих элементов в скобках. Например: $p=left( 1;3;2 right)$ или $p=left( 2;5;1;4;3 right)$. Буква может быть любой, но пусть будет $p$.:)

Далее для простоты изложения будем работать с перестановками длины 5 — они уже достаточно серьёзны для наблюдения всяких подозрительных эффектов, но ещё не настолько суровы для неокрепшего мозга, как перестановки длины 6 и более. Вот примеры таких перестановок:

[begin{align} & {{p}_{1}}=left( 1;2;3;4;5 right) \ & {{p}_{2}}=left( 1;3;2;5;4 right) \ & {{p}_{3}}=left( 5;4;3;2;1 right) \end{align}]

Естественно, перестановку длины $n$ можно рассматривать как функцию, которая определена на множестве $left{ 1;2;…;n right}$ и биективно отображает это множество на себя же. Возвращаясь к только что записанным перестановкам ${{p}_{1}}$, ${{p}_{2}}$ и ${{p}_{3}}$, мы вполне законно можем написать:

[{{p}_{1}}left( 1 right)=1;{{p}_{2}}left( 3 right)=2;{{p}_{3}}left( 2 right)=4;]

Количество различных перестановок длины $n$ всегда ограничено и равно $n!$ — это легко доказуемый факт из комбинаторики. Например, если мы захотим выписать все перестановки длины 5, то мы весьма заколебёмся, поскольку таких перестановок будет

[n!=5!=1cdot 2cdot 3cdot 4cdot 5=120]

Одной из ключевых характеристик всякой перестановки является количество инверсий в ней.

Определение. Инверсия в перестановке $p=left( {{a}_{1}};{{a}_{2}};…;{{a}_{n}} right)$ — всякая пара $left( {{a}_{i}};{{a}_{j}} right)$ такая, что $i lt j$, но ${{a}_{i}} gt {{a}_{j}}$. Проще говоря, инверсия — это когда большее число стоит левее меньшего (не обязательно соседнего).

Мы будем обозначать через $Nleft( p right)$ количество инверсий в перестановке $p$, но будьте готовы встретиться и с другими обозначениями в разных учебниках и у разных авторов — единых стандартов тут нет. Тема инверсий весьма обширна, и ей будет посвящён отдельный урок. Сейчас же наша задача — просто научиться считать их в реальных задачах.

Например, посчитаем количество инверсий в перестановке $p=left( 1;4;5;3;2 right)$:

[left( 4;3 right);left( 4;2 right);left( 5;3 right);left( 5;2 right);left( 3;2 right).]

Таким образом, $Nleft( p right)=5$. Как видите, ничего страшного в этом нет. Сразу скажу: дальше нас будет интересовать не столько само число $Nleft( p right)$, сколько его чётность/ нечётность. И тут мы плавно переходим к ключевому термину сегодняшнего урока.

Что такое определитель

Пусть дана квадратная матрица $A=left[ ntimes n right]$. Тогда:

Определение. Определитель матрицы $A=left[ ntimes n right]$ — это алгебраическая сумма $n!$ слагаемых, составленных следующим образом. Каждое слагаемое — это произведение $n$ элементов матрицы, взятых по одному из каждой строки и каждого столбца, умноженное на (−1) в степени количество инверсий:

[left| A right|=sumlimits_{n!}{{{left( -1 right)}^{Nleft( p right)}}cdot {{a}_{1;pleft( 1 right)}}cdot {{a}_{2;pleft( 2 right)}}cdot …cdot {{a}_{n;pleft( n right)}}}]

Принципиальным моментом при выборе множителей для каждого слагаемого в определителе является тот факт, что никакие два множителя не стоят в одной строчке или в одном столбце.

Благодаря этому можно без ограничения общности считать, что индексы $i$ множителей ${{a}_{i;j}}$ «пробегают» значения 1, …, $n$, а индексы $j$ являются некоторой перестановкой от первых:

[j=pleft( i right),quad i=1,2,…,n]

А когда есть перестановка $p$, мы легко посчитаем инверсии $Nleft( p right)$ — и очередное слагаемое определителя готово.

Естественно, никто не запрещает поменять местами множители в каком-либо слагаемом (или во всех сразу — чего мелочиться-то?), и тогда первые индексы тоже будут представлять собой некоторую перестановку. Но в итоге ничего не поменяется: суммарное количество инверсий в индексах $i$ и $j$ сохраняет чётность при подобных извращениях, что вполне соответствует старому-доброму правилу:

От перестановки множителей произведение чисел не меняется.

Вот только не надо приплетать это правило к умножению матриц — в отличие от умножения чисел, оно не коммутативно. Но это я отвлёкся.:)

Матрица 2×2

Вообще-то можно рассмотреть и матрицу 1×1 — это будет одна клетка, и её определитель, как нетрудно догадаться, равен числу, записанному в этой клетке. Ничего интересного.

Поэтому давайте рассмотрим квадратную матрицу размером 2×2:

[left[ begin{matrix} {{a}_{11}} & {{a}_{12}} \ {{a}_{21}} & {{a}_{22}} \end{matrix} right]]

Поскольку количество строк в ней $n=2$, то определитель будет содержать $n!=2!=1cdot 2=2$ слагаемых. Выпишем их:

[begin{align} & {{left( -1 right)}^{Nleft( 1;2 right)}}cdot {{a}_{11}}cdot {{a}_{22}}={{left( -1 right)}^{0}}cdot {{a}_{11}}cdot {{a}_{22}}={{a}_{11}}{{a}_{22}}; \ & {{left( -1 right)}^{Nleft( 2;1 right)}}cdot {{a}_{12}}cdot {{a}_{21}}={{left( -1 right)}^{1}}cdot {{a}_{12}}cdot {{a}_{21}}={{a}_{12}}{{a}_{21}}. \end{align}]

Очевидно, что в перестановке $left( 1;2 right)$, состоящей из двух элементов, нет инверсий, поэтому $Nleft( 1;2 right)=0$. А вот в перестановке $left( 2;1 right)$ одна инверсия имеется (собственно, 2 < 1), поэтому $Nleft( 2;1 right)=1.$

Итого универсальная формула вычисления определителя для матрицы 2×2 выглядит так:

[left| begin{matrix} {{a}_{11}} & {{a}_{12}} \ {{a}_{21}} & {{a}_{22}} \end{matrix} right|={{a}_{11}}{{a}_{22}}-{{a}_{12}}{{a}_{21}}]

Графически это можно представить как произведение элементов, стоящих на главной диагонали, минус произведение элементов на побочной:

Определитель матрицы 2×2

Рассмотрим пару примеров:

Задача. Вычислите определитель:

[left| begin{matrix} 5 & 6 \ 8 & 9 \end{matrix} right|;quad left| begin{matrix} 7 & 12 \ 14 & 1 \end{matrix} right|.]

Решение. Всё считается в одну строчку. Первая матрица:

[5cdot 9-8cdot 6=45-48=-3]

И вторая:

[7cdot 1-14cdot 12=7-168=-161]

Ответ: −3; −161.

Впрочем, это было слишком просто. Давайте рассмотрим матрицы 3×3 — там уже интересно.

Матрица 3×3

Теперь рассмотрим квадратную матрицу размера 3×3:

[left[ begin{matrix} {{a}_{11}} & {{a}_{12}} & {{a}_{13}} \ {{a}_{21}} & {{a}_{22}} & {{a}_{23}} \ {{a}_{31}} & {{a}_{32}} & {{a}_{33}} \end{matrix} right]]

При вычислении её определителя мы получим $3!=1cdot 2cdot 3=6$ слагаемых — ещё не слишком много для паники, но уже достаточно, чтобы начать искать какие-то закономерности. Для начала выпишем все перестановки из трёх элементов и посчитаем инверсии в каждой из них:

[begin{align} & {{p}_{1}}=left( 1;2;3 right)Rightarrow Nleft( {{p}_{1}} right)=Nleft( 1;2;3 right)=0; \ & {{p}_{2}}=left( 1;3;2 right)Rightarrow Nleft( {{p}_{2}} right)=Nleft( 1;3;2 right)=1; \ & {{p}_{3}}=left( 2;1;3 right)Rightarrow Nleft( {{p}_{3}} right)=Nleft( 2;1;3 right)=1; \ & {{p}_{4}}=left( 2;3;1 right)Rightarrow Nleft( {{p}_{4}} right)=Nleft( 2;3;1 right)=2; \ & {{p}_{5}}=left( 3;1;2 right)Rightarrow Nleft( {{p}_{5}} right)=Nleft( 3;1;2 right)=2; \ & {{p}_{6}}=left( 3;2;1 right)Rightarrow Nleft( {{p}_{6}} right)=Nleft( 3;2;1 right)=3. \end{align}]

Как и предполагалось, всего выписано 6 перестановок ${{p}_{1}}$, … ${{p}_{6}}$ (естественно, можно было бы выписать их в другой последовательности — суть от этого не изменится), а количество инверсий в них меняется от 0 до 3.

В общем, у нас будет три слагаемых с «плюсом» (там, где $Nleft( p right)$ — чётное) и ещё три с «минусом». А в целом определитель будет считаться по формуле:

[left| begin{matrix} {{a}_{11}} & {{a}_{12}} & {{a}_{13}} \ {{a}_{21}} & {{a}_{22}} & {{a}_{23}} \ {{a}_{31}} & {{a}_{32}} & {{a}_{33}} \end{matrix} right|=begin{matrix} {{a}_{11}}{{a}_{22}}{{a}_{33}}+{{a}_{12}}{{a}_{23}}{{a}_{31}}+{{a}_{13}}{{a}_{21}}{{a}_{32}}- \ -{{a}_{13}}{{a}_{22}}{{a}_{31}}-{{a}_{12}}{{a}_{21}}{{a}_{33}}-{{a}_{11}}{{a}_{23}}{{a}_{32}} \end{matrix}]

Вот только не надо сейчас садиться и яростно зубрить все эти индексы! Вместо непонятных цифр лучше запомните следующее мнемоническое правило:

Правило треугольника. Для нахождения определителя матрицы 3×3 нужно сложить три произведения элементов, стоящих на главной диагонали и в вершинах равнобедренных треугольников со стороной, параллельной этой диагонали, а затем вычесть такие же три произведения, но на побочной диагонали. Схематически это выглядит так:

Определитель матрицы 3×3: правило треугольников

Именно эти треугольники (или пентаграммы — кому как больше нравится) любят рисовать во всяких учебниках и методичках по алгебре. Впрочем, не будем о грустном. Давайте лучше посчитаем один такой определитель — для разминки перед настоящей жестью.:)

Задача. Вычислите определитель:

[left| begin{matrix} 1 & 2 & 3 \ 4 & 5 & 6 \ 7 & 8 & 1 \end{matrix} right|]

Решение. Работаем по правилу треугольников. Сначала посчитаем три слагаемых, составленных из элементов на главной диагонали и параллельно ей:

[begin{align} & 1cdot 5cdot 1+2cdot 6cdot 7+3cdot 4cdot 8= \ & =5+84+96=185 \end{align}]

Теперь разбираемся с побочной диагональю:

[begin{align} & 3cdot 5cdot 7+2cdot 4cdot 1+1cdot 6cdot 8= \ & =105+8+48=161 \end{align}]

Осталось лишь вычесть из первого числа второе — и мы получим ответ:

[185-161=24]

Вот и всё!

Ответ: 24.

Тем не менее, определители матриц 3×3 — это ещё не вершина мастерства. Самое интересное ждёт нас дальше.:)

Общая схема вычисления определителей

Как мы знаем, с ростом размерности матрицы $n$ количество слагаемых в определителе составляет $n!$ и быстро растёт. Всё-таки факториал — это вам не хрен собачий довольно быстро растущая функция.

Уже для матриц 4×4 считать определители напролом (т.е. через перестановки) становится как-то не оч. Про 5×5 и более вообще молчу. Поэтому к делу подключаются некоторые свойства определителя, но для их понимания нужна небольшая теоретическая подготовка.

Готовы? Поехали!

Что такое минор матрицы

Пусть дана произвольная матрица $A=left[ mtimes n right]$. Заметьте: не обязательно квадратная. В отличие от определителей, миноры — это такие няшки, которые существуют не только в суровых квадратных матрицах. Выберем в этой матрице несколько (например, $k$) строк и столбцов, причём $1le kle m$ и $1le kle n$. Тогда:

Определение. Минор порядка $k$ — определитель квадратной матрицы, возникающей на пересечении выбранных $k$ столбцов и строк. Также минором мы будем называть и саму эту новую матрицу.

Обозначается такой минор ${{M}_{k}}$. Естественно, у одной матрицы может быть целая куча миноров порядка $k$. Вот пример минора порядка 2 для матрицы $left[ 5times 6 right]$:

Выбор $k = 2$ столбцов и строк для формирования минора

Совершенно необязательно, чтобы выбранные строки и столбцы стояли рядом, как в рассмотренном примере. Главное, чтобы количество выбранных строк и столбцов было одинаковым (это и есть число $k$).

Есть и другое определение. Возможно, кому-то оно больше придётся по душе:

Определение. Пусть дана прямоугольная матрица $A=left[ mtimes n right]$. Если после вычеркивания в ней одного или нескольких столбцов и одной или нескольких строк образуется квадратная матрица размера $left[ ktimes k right]$, то её определитель — это и есть минор ${{M}_{k}}$. Саму матрицу мы тоже иногда будем называть минором — это будет ясно из контекста.

Как говорил мой кот, иногда лучше один раз навернуться с 11-го этажа есть корм, чем мяукать, сидя на балконе.

Пример. Пусть дана матрица

[A=left[ begin{matrix} begin{matrix} 1 \ 2 \ 3 \end{matrix} & begin{matrix} 7 \ 4 \ 0 \end{matrix} & begin{matrix} 9 \ 5 \ 6 \end{matrix} & begin{matrix} 0 \ 3 \ 1 \end{matrix} \end{matrix} right]]

Выбирая строку 1 и столбец 2, получаем минор первого порядка:

[{{M}_{1}}=left| 7 right|=7]

Выбирая строки 2, 3 и столбцы 3, 4, получаем минор второго порядка:

[{{M}_{2}}=left| begin{matrix} 5 & 3 \ 6 & 1 \end{matrix} right|=5-18=-13]

А если выбрать все три строки, а также столбцы 1, 2, 4, будет минор третьего порядка:

[{{M}_{3}}=left| begin{matrix} 1 & 7 & 0 \ 2 & 4 & 3 \ 3 & 0 & 1 \end{matrix} right|]

Считать этот определитель мне уже в лом. Но он равен 53.:)

Читателю не составит труда найти и другие миноры порядков 1, 2 или 3. Поэтому идём дальше.

Алгебраические дополнения

«Ну ok, и что дают нам эти миньоны миноры?» — наверняка спросите вы. Сами по себе — ничего. Но в квадратных матрицах у каждого минора появляется «компаньон» — дополнительный минор, а также алгебраическое дополнение. И вместе эти два ушлёпка позволят нам щёлкать определители как орешки.

Определение. Пусть дана квадратная матрица $A=left[ ntimes n right]$, в которой выбран минор ${{M}_{k}}$. Тогда дополнительный минор для минора ${{M}_{k}}$ — это кусок исходной матрицы $A$, который останется при вычёркивании всех строк и столбцов, задействованных при составлении минора ${{M}_{k}}$:

Дополнительный минор к минору ${{M}_{2}}$

Уточним один момент: дополнительный минор — это не просто «кусок матрицы», а определитель этого куска.

Обозначаются дополнительные миноры с помощью «звёздочки»: $M_{k}^{*}$:

[M_{k}^{*}=left| Anabla {{M}_{k}} right|]

где операция $Anabla {{M}_{k}}$ буквально означает «вычеркнуть из $A$ строки и столбцы, входящие в ${{M}_{k}}$». Эта операция не является общепринятой в математике — я её сам только что придумал для красоты повествования.:)

Дополнительные миноры редко используются сами по себе. Они являются частью более сложной конструкции — алгебраического дополнения.

Определение. Алгебраическое дополнение минора ${{M}_{k}}$ — это дополнительный минор $M_{k}^{*}$, умноженный на величину ${{left( -1 right)}^{S}}$, где $S$ — сумма номеров всех строк и столбцов, задействованных в исходном миноре ${{M}_{k}}$.

Как правило, алгебраическое дополнение минора ${{M}_{k}}$ обозначается через ${{A}_{k}}$. Поэтому:

[{{A}_{k}}={{left( -1 right)}^{S}}cdot M_{k}^{*}]

Сложно? На первый взгляд — да. Но это не точно. Потому что на самом деле всё легко. Рассмотрим пример:

Пример. Дана матрица 4×4:

[A=left[ begin{matrix} 1 & 2 & 3 & 4 \ 5 & 6 & 7 & 8 \ 9 & 10 & 11 & 12 \ 13 & 14 & 15 & 16 \end{matrix} right]]

Выберем минор второго порядка

[{{M}_{2}}=left| begin{matrix} 3 & 4 \ 15 & 16 \end{matrix} right|]

Капитан Очевидность как бы намекает нам, что при составлении этого минора были задействованы строки 1 и 4, а также столбцы 3 и 4. Вычёркиваем их — получим дополнительный минор:

[M_{2}^{*}=left| begin{matrix} 5 & 6 \ 9 & 10 \end{matrix} right|=50-54=-4]

Осталось найти число $S$ и получить алгебраическое дополнение. Поскольку мы знаем номера задействованных строк (1 и 4) и столбцов (3 и 4), всё просто:

[begin{align} & S=1+4+3+4=12; \ & {{A}_{2}}={{left( -1 right)}^{S}}cdot M_{2}^{*}={{left( -1 right)}^{12}}cdot left( -4 right)=-4end{align}]

Ответ: ${{A}_{2}}=-4$

Вот и всё! По сути, всё различие между дополнительным минором и алгебраическим дополнением — только в минусе спереди, да и то не всегда.

Наша задача сейчас — научиться быстро считать алгебраические дополнения, потому что они являются составной частью «Теоремы, Которую Нельзя Называть». Но мы всё же назовём. Встречайте:

Теорема Лапласа

И вот мы пришли к тому, зачем, собственно, все эти миноры и алгебраические дополнения были нужны.

Теорема Лапласа о разложении определителя. Пусть в матрице размера $left[ ntimes n right]$ выбрано $k$ строк (столбцов), причём $1le kle n-1$. Тогда определитель этой матрицы равен сумме всех произведений миноров порядка $k$, содержащихся в выбранных строках (столбцах), на их алгебраические дополнения:

[left| A right|=sum{{{M}_{k}}cdot {{A}_{k}}}]

Причём таких слагаемых будет ровно $C_{n}^{k}$.

Ладно, ладно: про $C_{n}^{k}$ — это я уже понтуюсь, в оригинальной теореме Лапласа ничего такого не было. Но комбинаторику никто не отменял, и буквально беглый взгляд на условие позволит вам самостоятельно убедиться, что слагаемых будет именно столько.:)

Мы не будем её доказывать, хоть это и не представляет особой трудности — все выкладки сводятся к старым-добрым перестановкам и чётности/ нечётности инверсий. Тем не менее, доказательство будет представлено в отдельном параграфе, а сегодня у нас сугубо практический урок.

Поэтому переходим к частному случаю этой теоремы, когда миноры представляют собой отдельные клетки матрицы.

Разложение определителя по строке и столбцу

То, о чём сейчас пойдёт речь — как раз и есть основной инструмент работы с определителями, ради которого затевались вся эта дичь с перестановками, минорами и алгебраическими дополнениями.

Читайте и наслаждайтесь:

Следствие из Теоремы Лапласа (разложение определителя по строке/столбцу). Пусть в матрице размера $left[ ntimes n right]$ выбрана одна строка. Минорами в этой строке будут $n$ отдельных клеток:

[{{M}_{1}}={{a}_{ij}},quad j=1,…,n]

Дополнительные миноры тоже легко считаются: просто берём исходную матрицу и вычёркиваем строку и столбец, содержащие ${{a}_{ij}}$. Назовём такие миноры $M_{ij}^{*}$.

Для алгебраического дополнения ещё нужно число $S$, но в случае с минором порядка 1 это просто сумма «координат» клетки ${{a}_{ij}}$:

[S=i+j]

И тогда исходный определитель можно расписать через ${{a}_{ij}}$ и $M_{ij}^{*}$ согласно теореме Лапласа:

[left| A right|=sumlimits_{j=1}^{n}{{{a}_{ij}}cdot {{left( -1 right)}^{i+j}}cdot {{M}_{ij}}}]

Это и есть формула разложения определителя по строке. Но то же верно и для столбцов.

Из этого следствия можно сразу сформулировать несколько выводов:

  1. Эта схема одинаково хорошо работает как для строк, так и для столбцов. На самом деле чаще всего разложение будет идти именно по столбцам, нежели по строкам.
  2. Количество слагаемых в разложении всегда ровно $n$. Это существенно меньше $C_{n}^{k}$ и уж тем более $n!$.
  3. Вместо одного определителя $left[ ntimes n right]$ придётся считать несколько определителей размера на единицу меньше: $left[ left( n-1 right)times left( n-1 right) right]$.

Последний факт особенно важен. Например, вместо зверского определителя 4×4 теперь достаточно будет посчитать несколько определителей 3×3 — с ними мы уж как-нибудь справимся.:)

Что ж, попробуем посчитать одну такую задачку?

Задача. Найдите определитель:

[left| begin{matrix} 1 & 2 & 3 \ 4 & 5 & 6 \ 7 & 8 & 9 \end{matrix} right|]

Решение. Разложим этот определитель по первой строке:

[begin{align} left| A right|=1cdot {{left( -1 right)}^{1+1}}cdot left| begin{matrix} 5 & 6 \ 8 & 9 \end{matrix} right|+ & \ 2cdot {{left( -1 right)}^{1+2}}cdot left| begin{matrix} 4 & 6 \ 7 & 9 \end{matrix} right|+ & \ 3cdot {{left( -1 right)}^{1+3}}cdot left| begin{matrix} 4 & 5 \ 7 & 8 \end{matrix} right|= & \end{align}]

[begin{align} & =1cdot left( 45-48 right)-2cdot left( 36-42 right)+3cdot left( 32-35 right)= \ & =1cdot left( -3 right)-2cdot left( -6 right)+3cdot left( -3 right)=0. \end{align}]

Ответ: 0.

Задача. Найдите определитель:

[left| begin{matrix} 0 & 1 & 1 & 0 \ 1 & 0 & 1 & 1 \ 1 & 1 & 0 & 1 \ 1 & 1 & 1 & 0 \end{matrix} right|]

Решение. Для разнообразия давайте в этот раз работать со столбцами. Например, в последнем столбце присутствуют сразу два нуля — очевидно, это значительно сократит вычисления. Сейчас увидите почему.

Итак, раскладываем определитель по четвёртому столбцу:

[begin{align} left| begin{matrix} 0 & 1 & 1 & 0 \ 1 & 0 & 1 & 1 \ 1 & 1 & 0 & 1 \ 1 & 1 & 1 & 0 \end{matrix} right|=0cdot {{left( -1 right)}^{1+4}}cdot left| begin{matrix} 1 & 0 & 1 \ 1 & 1 & 0 \ 1 & 1 & 1 \end{matrix} right|+ & \ +1cdot {{left( -1 right)}^{2+4}}cdot left| begin{matrix} 0 & 1 & 1 \ 1 & 1 & 0 \ 1 & 1 & 1 \end{matrix} right|+ & \ +1cdot {{left( -1 right)}^{3+4}}cdot left| begin{matrix} 0 & 1 & 1 \ 1 & 0 & 1 \ 1 & 1 & 1 \end{matrix} right|+ & \ +0cdot {{left( -1 right)}^{4+4}}cdot left| begin{matrix} 0 & 1 & 1 \ 1 & 0 & 1 \ 1 & 1 & 0 \end{matrix} right| & \end{align}]

И тут — о, чудо! — два слагаемых сразу улетают коту под хвост, поскольку в них есть множитель «0». Остаётся ещё два определителя 3×3, с которыми мы легко разберёмся:

[begin{align} & left| begin{matrix} 0 & 1 & 1 \ 1 & 1 & 0 \ 1 & 1 & 1 \end{matrix} right|=0+0+1-1-1-0=-1; \ & left| begin{matrix} 0 & 1 & 1 \ 1 & 0 & 1 \ 1 & 1 & 1 \end{matrix} right|=0+1+1-0-0-1=1. \end{align}]

Возвращаемся к исходнику и находим ответ:

[left| begin{matrix} 0 & 1 & 1 & 0 \ 1 & 0 & 1 & 1 \ 1 & 1 & 0 & 1 \ 1 & 1 & 1 & 0 \end{matrix} right|=1cdot left( -1 right)+left( -1 right)cdot 1=-2]

Ну вот и всё. И никаких 4! = 24 слагаемых считать не пришлось.:)

Ответ: −2

Основные свойства определителя

В последней задаче мы видели, как наличие нулей в строках (столбцах) матрицы резко упрощает разложение определителя и вообще все вычисления. Возникает естественный вопрос: а нельзя ли сделать так, чтобы эти нули появились даже в той матрице, где их изначально не было?

Ответ однозначен: можно. И здесь нам на помощь приходят свойства определителя:

  1. Если поменять две строчки (столбца) местами, определитель поменяет знак;
  2. Если одну строку (столбец) умножить на число $k$, то весь определитель тоже умножится на число $k$;
  3. Если взять одну строку и прибавить (вычесть) её сколько угодно раз из другой, определитель не изменится;
  4. Если две строки определителя одинаковы, либо пропорциональны, либо одна из строк заполнена нулями, то весь определитель равен нулю;
  5. Все указанные выше свойства верны и для столбцов.
  6. При транспонировании матрицы определитель не меняется;
  7. Определитель произведения матриц равен произведению определителей.

Особую ценность представляет третье свойство: мы можем вычитать из одной строки (столбца) другую до тех пор, пока в нужных местах не появятся нули.

Чаще всего расчёты сводится к тому, чтобы «обнулить» весь столбец везде, кроме одного элемента, а затем разложить определитель по этому столбцу, получив матрицу размером на 1 меньше.

Давайте посмотрим, как это работает на практике:

Задача. Найдите определитель:

[left| begin{matrix} 1 & 2 & 3 & 4 \ 4 & 1 & 2 & 3 \ 3 & 4 & 1 & 2 \ 2 & 3 & 4 & 1 \end{matrix} right|]

Решение. Нулей тут как бы вообще не наблюдается, поэтому можно «долбить» по любой строке или столбцу — объём вычислений будет примерно одинаковым. Давайте не будем мелочиться и «обнулим» первый столбец: в нём уже есть клетка с единицей, поэтому просто возьмём первую строчку и вычтем её 4 раза из второй, 3 раза из третьей и 2 раза из последней.

В результате мы получим новую матрицу, но её определитель будет тем же:

[begin{matrix} left| begin{matrix} 1 & 2 & 3 & 4 \ 4 & 1 & 2 & 3 \ 3 & 4 & 1 & 2 \ 2 & 3 & 4 & 1 \end{matrix} right|begin{matrix} downarrow \ -4 \ -3 \ -2 \end{matrix}= \ =left| begin{matrix} 1 & 2 & 3 & 4 \ 4-4cdot 1 & 1-4cdot 2 & 2-4cdot 3 & 3-4cdot 4 \ 3-3cdot 1 & 4-3cdot 2 & 1-3cdot 3 & 2-3cdot 4 \ 2-2cdot 1 & 3-2cdot 2 & 4-2cdot 3 & 1-2cdot 4 \end{matrix} right|= \ =left| begin{matrix} 1 & 2 & 3 & 4 \ 0 & -7 & -10 & -13 \ 0 & -2 & -8 & -10 \ 0 & -1 & -2 & -7 \end{matrix} right| \end{matrix}]

Теперь с невозмутимостью Пятачка раскладываем этот определитель по первому столбцу:

[begin{matrix} 1cdot {{left( -1 right)}^{1+1}}cdot left| begin{matrix} -7 & -10 & -13 \ -2 & -8 & -10 \ -1 & -2 & -7 \end{matrix} right|+0cdot {{left( -1 right)}^{2+1}}cdot left| … right|+ \ +0cdot {{left( -1 right)}^{3+1}}cdot left| … right|+0cdot {{left( -1 right)}^{4+1}}cdot left| … right| \end{matrix}]

Понятно, что «выживет» только первое слагаемое — в остальных я даже определители не выписывал, поскольку они всё равно умножаются на ноль. Коэффициент перед определителем равен единице, т.е. его можно не записывать.

Зато можно вынести «минусы» из всех трёх строк определителя. По сути, мы трижды вынесли множитель (−1):

[left| begin{matrix} -7 & -10 & -13 \ -2 & -8 & -10 \ -1 & -2 & -7 \end{matrix} right|=cdot left| begin{matrix} 7 & 10 & 13 \ 2 & 8 & 10 \ 1 & 2 & 7 \end{matrix} right|]

Получили мелкий определитель 3×3, который уже можно посчитать по правилу треугольников. Но мы попробуем разложить и его по первому столбцу — благо в последней строчке гордо стоит единица:

[begin{align} & left( -1 right)cdot left| begin{matrix} 7 & 10 & 13 \ 2 & 8 & 10 \ 1 & 2 & 7 \end{matrix} right|begin{matrix} -7 \ -2 \ uparrow \end{matrix}=left( -1 right)cdot left| begin{matrix} 0 & -4 & -36 \ 0 & 4 & -4 \ 1 & 2 & 7 \end{matrix} right|= \ & =cdot left| begin{matrix} -4 & -36 \ 4 & -4 \end{matrix} right|=left( -1 right)cdot left| begin{matrix} -4 & -36 \ 4 & -4 \end{matrix} right| \end{align}]

Можно, конечно, ещё поприкалываться и разложить матрицу 2×2 по строке (столбцу), но мы же с вами адекватны, поэтому просто посчитаем ответ:

[left( -1 right)cdot left| begin{matrix} -4 & -36 \ 4 & -4 \end{matrix} right|=left( -1 right)cdot left( 16+144 right)=-160]

Вот так и разбиваются мечты. Всего-то −160 в ответе.:)

Ответ: −160.

Парочка замечаний перед тем, как мы перейдём к последней задаче:

  1. Исходная матрица была симметрична относительно побочной диагонали. Все миноры в разложении тоже симметричны относительно той же побочной диагонали.
  2. Строго говоря, мы могли вообще ничего не раскладывать, а просто привести матрицу к верхнетреугольному виду, когда под главной диагональю стоят сплошные нули. Тогда (в точном соответствии с геометрической интерпретацией, кстати) определитель равен произведению ${{a}_{ii}}$ — чисел на главной диагонали.

Идём дальше. Последняя задача в сегодняшнем уроке.

Задача. Найдите определитель:

[left| begin{matrix} 1 & 1 & 1 & 1 \ 2 & 4 & 8 & 16 \ 3 & 9 & 27 & 81 \ 5 & 25 & 125 & 625 \end{matrix} right|]

Решение. Ну, тут первая строка прямо-таки напрашивается на «обнуление». Берём первый столбец и вычитаем ровно один раз из всех остальных:

[begin{align} & left| begin{matrix} 1 & 1 & 1 & 1 \ 2 & 4 & 8 & 16 \ 3 & 9 & 27 & 81 \ 5 & 25 & 125 & 625 \end{matrix} right|= \ & =left| begin{matrix} 1 & 1-1 & 1-1 & 1-1 \ 2 & 4-2 & 8-2 & 16-2 \ 3 & 9-3 & 27-3 & 81-3 \ 5 & 25-5 & 125-5 & 625-5 \end{matrix} right|= \ & =left| begin{matrix} 1 & 0 & 0 & 0 \ 2 & 2 & 6 & 14 \ 3 & 6 & 24 & 78 \ 5 & 20 & 120 & 620 \end{matrix} right| \end{align}]

Раскладываем по первой строке, а затем выносим общие множители из оставшихся строк:

[cdot left| begin{matrix} 2 & 6 & 14 \ 6 & 24 & 78 \ 20 & 120 & 620 \end{matrix} right|=cdot left| begin{matrix} 1 & 3 & 7 \ 1 & 4 & 13 \ 1 & 6 & 31 \end{matrix} right|]

Снова наблюдаем «красивые» числа, но уже в первом столбце — раскладываем определитель по нему:

[begin{align} & 240cdot left| begin{matrix} 1 & 3 & 7 \ 1 & 4 & 13 \ 1 & 6 & 31 \end{matrix} right|begin{matrix} downarrow \ -1 \ -1 \end{matrix}=240cdot left| begin{matrix} 1 & 3 & 7 \ 0 & 1 & 6 \ 0 & 3 & 24 \end{matrix} right|= \ & =240cdot {{left( -1 right)}^{1+1}}cdot left| begin{matrix} 1 & 6 \ 3 & 24 \end{matrix} right|= \ & =240cdot 1cdot left( 24-18 right)=1440 \end{align}]

Порядок. Задача решена.

Ответ: 1440

Всё. Хорош читать этот бред.:)

Смотрите также:

  1. Обратная матрица
  2. Умножение матриц
  3. Геометрическая вероятность
  4. Решение задач B12: №448—455
  5. Задачи на проценты: формула, упрощающая вычисления
  6. Задача B4 про три дороги — стандартная задача на движение

Часто в ВУЗе попадаются задачи по высшей математики, в которых необходимо вычислить определитель матрицы. К слову, определитель может быть только в квадратных матрицах. Ниже рассмотрим основные определения, какими свойствами обладает определитель и как его правильно вычислить.. Также на примерах покажем подробное решение.

Что такое определитель матрицы: вычисление определителя при помощи определения

Определитель матрицы

A = begin{pmatrix}  a_1_1&a_1_2\  a_2_1&a_2_2  end{pmatrix}right второго порядка – это число |A| = a_{11}a_{22} - a_{12}a_{21}.

Определитель матрицы обозначается det A – (сокращенно от латинского названия детерминант), или |A|.

Если:A = begin{pmatrix}  a_1_1&a_1_2\  a_2_1&a_2_2  end{pmatrix}right, тогда получается |A| = det A = begin{pmatrix}  a_1_1&a_1_2\  a_2_1&a_2_2  end{pmatrix}right

Напомним ещё несколько вспомогательных определений:

Упорядоченный набор чисел, который состоит из n элементов называется перестановкой порядка n.

Для множества, которое содержит n элементов есть факториал (n), который всегда обозначается восклицательным знаком: n!. Перестановки отличаются друг от друга всего лишь порядком следования. Чтобы вам было понятнее, приведём пример:

Рассмотрим множество из трёх элементов {3, 6, 7}. Всего перестановок 6, так как 3! = 1 * 2 * 3 = 6.:

1. 3, 6, 7;

2. 3, 7, 6;

3. 6, 3, 7;

4. 6, 7, 3;

5. 7, 3, 6;

6. 7, 6, 3

Инверсия в перестановке порядка n – это упорядоченный набор чисел (его ещё называют биекцией), где из них два числа образуют некий беспорядок. Это когда большее из чисел в данной перестановке расположено левее меньшего числа. 

Выше мы рассматривали пример с инверсией перестановки, где были числа {6, 3, 7}. Так вот, возьмём вторую строку, где судя по данным числам получается, что q = 2, а k = 3, так как второй элемент 7 больше третьего элемента 6. Возьмём для сравнения шестую строку, где расположены числа: 7, 6, 3. Здесь есть три пары: p = 1, а k = 2, так как 7 > 6; p = 1, k = 3, так как 7 > 3; p = 2, k = 36 > 3.

Саму инверсию мы изучать не будем, а вот перестановки нам очень пригодятся в дальнейшем рассмотрении темы.

Определитель матрицы nxn – число:

det (A) = sum_{a_1, a_2,..., a_n}limits (-1)^{N(a_1, a_2,..., a_n)}* a_alpha_1_1*a_alpha_2_2...*a_alpha_n_n, где

alpha_1, alpha_2, alpha_n – перестановка чисел от 1 до бесконечного числа (n), а {N(a_1, a_2,..., a_n)} – число инверсий в перестановке. Таким образом, в определитель входит n! слагаемых, которые называются “членами определителя”.

Можно вычислять определитель матрицы второго порядка, третьего и даже четвёртого. Также стоит упомянуть: 

определитель матрицы |A| – это число, которое равняется sum_{k = 1}^{n!}limits(-1)^{N_k} * a_{1j1(k)} * a_{2j2(k)} * ...*a_{njn(k)}.

Чтобы понять данную формулу, опишем её более подробно. Определитель квадратной матрицы n x n – это сумма, которая содержит n! слагаемых, а каждое слагаемое является собой произведением определённого количества (n) элементов матрицы. При этом, в каждом произведении есть элемент из каждой строки и каждого столбца матрицы.

Перед определённым слагаемым может появится (-1) в том случае, если элементы матрицы в произведении идут по порядку (по номеру строку), а количество инверсий N_k в перестановке k множество номеров столбцов нечётно.

Выше упоминалось о том, что определитель матрицы A обозначается |A| или det(A), то есть, определитель часто называют детерминантом.

Итак, вернёмся к формуле:

sum_{k = 1}^{n!}limits(-1)^{N_k} * a_{1j1(k)} * a_{2j2(k)} * ...*a_{njn(k)}.

Из формулы видно, что определитель матрицы первого порядка – это элемент этой же матрицы |a_{11}| = a_{11}.

Нужна помощь в написании работы?

Написание учебной работы за 1 день от 100 рублей. Посмотрите отзывы наших клиентов и узнайте стоимость вашей работы.

Подробнее

Вычисление определителя матрицы второго порядка

Чаще всего на практике определитель матрицы решается методами второго, третьего и реже, четвёртого порядка. Рассмотрим, как вычисляется определитель матрицы второго порядка:

A = begin{vmatrix} a_{11}&a_{12}\ a_{21}&a_{22} end{vmatrix} right

В матрице второго порядка n = 2, отсюда следует, что факториал n! = 2! = 2. Прежде чем применить формулу

A = sum_{k = 1}^{n!}limits(-1)^{N_k} * a_{1j1(k)} * a_{2j2(k)} * ...*a_{njn(k)}. необходимо определить, какие данные у нас получаются:

1. k = 2;

2. перестановки множеств: 1, 2 и 2, 1;

3. количество инверсий в перестановке N_k: 0 и 1, так как 2 > 1;

4. соответствующие произведения (-1)^{N_k} * a_{1j1(k)} * a_{2j2(k)}: a_{11} * a_{22} и -a_{12} * a_{21}.

Получается:

|A| = begin{vmatrix}  a_{11}&a_{12}\  a_{21}&a_{22}  end{vmatrix} = sum_{k = 1}^{n!}limits(-1)^{N_k} * a_{1j1(k)} * a_{2j2(k)} = a_{11} * a_{22} - a_{12} * a_{21}.  right

Исходя из вышесказанного мы получаем формулу для вычисления определителя квадратной матрицы второго порядка, то есть 2 x 2:

begin{vmatrix} a_{11}&a_{12}\ a_{21}&a_{22} end{vmatrix} = a_{11} * a_{22} - a_{12} * a_{21} right

Рассмотрим на конкретном примере, как вычислять определитель квадратной матрицы второго порядка:

Задача

Вычислить определитель матрицы 2 x 2:

A = begin{vmatrix} 2&3\ - 6&1 end{vmatrix}right

Решение

Итак, у нас получается a_{11} = 2, a_{12} = 3, a_{21} = - 6, a_{22} = 1.

Для решения необходимо воспользоваться ранее рассмотренной формулой:

A = begin{vmatrix} a_{11}&a_{12}\ a_{21}&a_{22} end{vmatrix} = a_{11} * a_{22} - a_{12} * a_{21} right

Подставляем числа с примера и находим:

A = begin{vmatrix} 2&3\ - 6&1 end{vmatrix} = 2 * 1 - 3 * (-6) = 3 - (-18) = 21 right

Ответ

Определитель матрицы второго порядка = 21.

Вычисление определителя матрицы третьего порядка: пример и решение по формуле

Определитель матрицы третьего порядка – это число, полученное из девяти заданных чисел, расположенных в виде квадратной таблицы,

Определитель третьего порядка находится почти так же, как и определитель второго порядка. Разница лишь в формуле. Поэтому, если хорошо ориентироваться в формуле, тогда и проблем с решением не будет.

Рассмотрим квадратную матрицу третьего порядка 3 * 3:

A = begin{vmatrix} a_{11}&a_{12}&a_{13}\ a_{21}&a_{22}&a_{23}\ a_{31}&a_{32}&a_{33} end{vmatrix} right

Исходя из данной матрицы, понимаем, что n = 3, соответственно, факториал (n) = 3, а это значит, что всего перестановок получается 6

Чтобы применить правильно формулу |A| =  sum_{k = 1}^{n!}limits(-1)^{N_k} * a_{1j_1(k)} * a_{2j_2(k)} ... * a_{nj_n(k), необходимо найти данные:

Итак, всего перестановок множества {1, 2, 3} = 6:

1. 1, 2, 3, количество инверсий в перестановке 0, а соответствующие произведения = a_{11} * a_{22} * a_{33};

2. 1, 3, 2, количество инверсий в перестановке 1 (3 >2), соответствующие произведения = -a_{11} * a_{23} * a_{32};

3. 2, 1, 3, инверсий в перестановке 1 (2 > 1), соответствующие произведение = -a_{12} * a_{21} * a_{33};

4. 2, 3, 1; инверсий в перестановке 2 (2 > 1; 3 > 1), соответствующие произведение = a_{12} * a_{23} * a_{31}

5. 3, 1, 2; инверсий в перестановке 2 (3 > 1; 3 > 2), соответствующие произведение = a_{13} * a_{21} * a_{32}

6. 3, 2, 1; инверсий в перестановке 3 (3 > 1; 3 > 2; 2 >1), соответствующие произведение = -a_{13} * a_{22} * a_{31}.

Теперь у нас получается:

A = begin{vmatrix} a_{11}&a_{12}&a_{13}\ a_{21}&a_{22}&a_{23}\ a_{31}&a_{32}&a_{33} end{vmatrix} = sum_{k = 1}^{n!}limits(-1)^{N_k} * a_{1j_1(k)} * a_{2j_2(k)} * a_{3j_3(k)} = a_{11} * a_{22} * a_{33} - a_{11} * a_{23} * a_{32} + a_{12} * a_{23} * a_{31} + a_{13} * a_{21} * a_{32} - a_{13} * a_{22} * a_{31} right

Таким образом у нас получена формула для вычисления определителя матрицы порядка 3 x 3:

begin{vmatrix} a_{11}&a_{12}&a_{13}\ a_{21}&a_{22}&a_{23}\ a_{31}&a_{32}&a_{33} end{vmatrix} right =  a_{11} * a_{22} * a_{33} + a_{12} * a_{23} * a_{31} + a_{13} * a_{21} * a_{32} -\- a_{13} * a_{22} * a_{31} - a_{12} * a_{21} * a_{33} - a_{11} * a_{23} * a_{32}.

Нахождение матрицы третьего порядка по правилу треугольника (правило Саррюса)

Как говорилось выше, элементы определителя 3-го порядка расположены в трёх строках и трёх столбцах. Если ввести обозначение общего элемента a_{ij}(i = 1, 2, 3; j = 1, 2, 3), тогда первый элемент обозначает номер строки, а второй элемент из индексов – номер столбца. Есть главная (элементы a_{11}, a_{22}, a_{33)) и побочная (элементы a_{31}, a_{22}, a_{13}) диагонали определителя. Слагаемые в правой части называются членами определителя).

Видно, что каждый член определителя находится в схеме только по одному элементу в каждой строке и каждого столбца.

Вычислять определитель можно при помощи правила прямоугольника, который изображён в виде схемы. Красным цветом выделены члены определителя из элементов главной диагонали, а также члены из элементов, которые находятся в  вершине треугольников, что имеют по одной стороне, параллельны главной диагонали (лева схема), беруться со знаком "+".

Члены с синими стрелками из элементов побочной диагонали, а также из элементов, которые находятся в вершинах треугольников, что имеют стороны, параллельные побочной диагонали (правая схема) берутся со знаком "-".

Правило треугольника

На следующем примере научимся, как вычислять определитель квадратной матрицы третьего порядка.

Задача

Вычислить определитель матрицы третьего порядка:

 begin{vmatrix} 1&4&-3\ 2&3&5\ -4&1&-1 end{vmatrix} right

Решение

В этом примере:

a_{11} = 1, a_{12} = 4, a_{13} = -3,

a_{21} = 2, a_{22} = 3, a_{23} = 5,

a_{31} = -4, a_{32} = 1, a_{33} = -1.

Вычисляем определитель, применяя формулу или схему, которые рассматривались выше:

 begin{vmatrix} 1&4&-3\ 2&3&5\ -4&1&-1 end{vmatrix} = a_{11} * a_{22} * a_{33} + a_{12} * a_{23} * a_{31} + a_{13} * a_{21} * a_{32} - a_{13} * a_{22} * a_{31} - a_{12} * a_{21} * a_{33} - a_{11} * a_{23} * a_{32} = 1 * 3 * (-1) + 4 * 5 * (-4) + (-3) * 2 * 1 - (-3) * 3 * (-4) - 4 * 2 * (-1) - 1 * 5 * 1 = -3 - 80 - 6 - 36 + 8 - 5 = -122 right

Ответ

Определитель матрицы третьего порядка = -122

Рекомендуем запомнить формулы для нахождения определителя матрицы второго и третьего порядка, так как они часто применяются на зачётах и экзаменах.

Основные свойства определителей матрицы третьего порядка

На основании предыдущих определений и формул рассмотрим основные свойства определителя матрицы.

1. Размер определителя не изменится при замене соответствующих строк, столбцов (такая замена называется транспонированием).

left begin{vmatrix} a_1_1&a_1_2&a_1_3\ a_2_1&a_2_2&a_2_3\ a_3_1&a_3_2&a_3_3 end{vmatrix} = begin{vmatrix}a_1_1&a_2_1&a_3_1\a_1_2&a_2_2&a_3_2\a_1_3&a_2_3&a_3_3end{vmatrix}right

На примере убедимся, что определитель матрицы A равен определителю транспонированной матрицы:

|A| = begin{vmatrix} 0&3&1\ 1&2&3\. 3&2&3 end{vmatrix} right

Вспомним формулу для вычисления определителя: a_{11} * a_{22} * a_{33} + a_{12} * a_{23} * a_{31} + a_{13} * a_{21} * a_{32} - a_{13} * a_{22} * a_{31} - a_{12} * a_{21} * a_{33} - a_{11} * a_{23} * a_{32} = 0 * 2 * 3 + 3 * 3 * 3 + 1 * 1 * 2 - 1 * 2 * 3 - 3 * 1 * 3 - 0 * 3 * 2 = 0 + 27 + 2 - (-6) - (-9) - 0 = 44

Транспонируем матрицу:

A^T = begin{vmatrix} 0&3&1\ 1&2&3\. 3&2&3 end{vmatrix}^T = begin{vmatrix} 0&1&3\ 3&2&2\. 1&3&3 end{vmatrix} right

Вычисляем определитель транспонированной матрицы:

A^T = begin{vmatrix} 0&1&3\ 3&2&2\. 1&3&3 end{vmatrix}^T = 0 * 2 * 3 + 1 * 2 * 1 + 3 * 3 * 3 - 3 * 2 * 1 - 1 * 3 * 3 - 0 * 2 * 3 = 0 + 2 + 27 - (-6) - (-9) - 0 = 44 right

Мы убедились, что определитель транспортированной матрицы равен исходной матрице, что говорит о правильном решении.

2. Знак определителя изменится на противоположный, если в нём поменять местами любые два его столбца или две строки.

Рассмотрим на примере:

Даны две матрицы третьего порядка (3 x 3):

 A = begin{vmatrix} 1&4&-3\ 2&3&5\ -4&1&-1 end{vmatrix} right

 B = begin{vmatrix} -4&1&-1\ 2&3&5\ 1&4&-3 end{vmatrix} right

Нужно показать, что определители данных матриц противоположные.

Решение

В матрице A и в матрице B поменялись строки (третья с первой, и с первой на третью). Согласно второму свойству определители двух матриц должны отличаться знаком. То есть, одна матрица с положительным знаком, а вторая – с отрицательным. давайте проверим данное свойство, применив формулу для вычисления определителя.

 |A| = begin{vmatrix} 1&4&-3\ 2&3&5\ -4&1&-1 end{vmatrix} = 1 * 3 * (-1) + 4 * 5 * (-4) + (-3) * 2 * 1 - (-3) * 3 * (-4) - 4 * 2 * (-1) - 1 * 5 * 1 = -3 - 80 - 6 - 36 + 8 - 5 = -122 right

 |B| = begin{vmatrix} -4&1&-1 2&3&5\ 1&4&-3 end{vmatrix} = (-4) * 3 * (-3) + 1 * 5 * 1 + (-1) * 2 * 4 - 1 * 3 * (-1) - 2 * 1 * (-3) - (-4) * 5 * 4 = 36 + 5 - 8 + 3 + 6 + 80 = 122 right

Свойство верно, так как - |A| = |B|.

3. Определитель равняется нулю, если в нём есть одинаковые соответствующие элементы в двух строках (столбцах). Пусть у определителя одинаковые элементы первого и второго столбцов:

Delta =begin{vmatrix} a&b&c\ b&b&a_2_3\ c&c&a_3_3 end{vmatrix}right

Поменяв местами одинаковые столбцы, мы, согласно свойству 2 получим новый определитель: Delta_1-Delta. С другой стороны, новый определитель совпадает с изначальным, поскольку одинаковые ответы элементы, то есть Delta_1Delta. Из этих равенств у нас получается: Delta_1-Deltalongrightarrow 2Delta=0longrightarrowDelta = 0.

4. Определитель равняется нулю, если все элементы одной строки (столбца) нули. Это утверждение выплывает из того, что у каждого члена определителя по формуле (1) есть по одному, и только по одному элементу с каждой строки (столбца), у которого одни нули.

Рассмотрим на примере:

Покажем, что определитель матрицы равен нулю:

 A = begin{vmatrix} 3&2&2\ -2&3&3\ 8&-2&-2 end{vmatrix} right

В нашей матрицы есть два одинаковых столбца (второй и третий), поэтому, исходя из данного свойства, определитель должен равняться нулю. Проверим:

 |A| = begin{vmatrix} 3&2&2\ -2&3&3\ 8&-2&-2 end{vmatrix} = 3 * 3 * (-2) + 2 * 3 * 8 + 2 * (-2) * (-2) - 2 * 3 * 8 - 2 * (-2) * (-2) - 3 * 3 * (-2) = (-18) + 48 + 8 - 48 - 8 + 18 = 0 right

И действительно, определитель матрицы с двумя одинаковыми столбцами равняется нулю.

5. Общий множитель элементов первой строки (столбца) можно вынести за знак определителя:

leftbegin{vmatrix}a_1_1&a_1_2&ka_1_3\a_2_1&a_2_2&ka_2_3\a_3_1&a_3_2&ka_3_3end{vmatrix} = kbegin{vmatrix}a_1_1&a_1_2&a_1_3\a_2_1&a_2_2&a_2_3\a_3_1&a_3_2&a_3_3end{vmatrix}right.

6. Если элементы одной строки или одного столбца определителя пропорциональны соответствующим элементам второй строки (столбца), тогда такой определитель равняется нулю.

Действительно, за свойством 5 коэффициент пропорциональности можно вынести за знак определителя, и тогда воспользоваться свойством 3.

7. Если каждый из элементов строк (столбцов) определителя является суммой двух слагаемых, то этот определитель можно подать в виде суммы соответствующих определителей:

begin{vmatrix} a_1_1&a_1_2&a_1_3 + a'_1_3\ a_2_1&a_2_2&a_2_3 + a'_2_3\ a_3_1&a_3_2&a_3_3 + a'_3_3 end{vmatrix} =begin{vmatrix} a_1_1&a_1_2&a_1_3\ a_2_1&a_2_2&a_2_3\ a_3_1&a_3_2&a_3_3end{vmatrix} +begin{vmatrix} a_1_1&a_1_2&a'_1_3\ a_2_1&a_2_2&a'_2_3\ a_3_1&a_3_2&a'_3_3 end{vmatrix} right.

Для проверки достаточно записать в развёрнутом виде по (1) определитель, что в левой части равенства, тогда отдельно сгруппировать члены, в которых содержатся элементы a_i_3 и a'_i_3 (i = 1, 2, 3).Каждая из полученных групп слагаемых будет соответственно первым и вторым определителем с правой части равенства.

8. Значения определения не изменятся, если к элементу одной строки или одного столбца прибавить соответствующие элементы второй строки (столбца), умноженные на одно и то же число:

begin{vmatrix} a_1_1&a_1_2&a_1_3\ a_2_1&a_2_2&a_2_3\ a_3_1&a_3_2&a_3_3 end{vmatrix} =begin{vmatrix} a_1_1&a_1_2&a_1_3 + ka_1_1\ a_2_1&a_2_2&a_2_3 + ka_2_1\ a_3_1&a_3_2&a_3_3 + ka_3_1end{vmatrix} right.

Это равенство получается исходя из свойств 6 и 7.

9. Определитель матрицы A = |a_{ij}|, i = 1, 2, 3, 4, ..., n, j = 1, 2, 3, 4, ..., n равняется сумме произведений элементов какой-либо строки или столбца на их алгебраические дополнения.

 begin{vmatrix} a_{11}&a_{12}&dots&a_{1n}\ a_{21}&a_{22}&dots&a_{2n}\ vdots&vdots&vdots&vdots\ a_{n1}&a_{n2}&dots&a_{nn} end{vmatrix} = a_{p1} * A_{p1} + a_{p2} * A_{p2} + dots + a_{pn} * A_{pn} =\ = a_{1q} * A_{1q} + a_{2q} * A_{2q} + dots + a_{nq} * A_{nq}. right

Здесь по A_{ij} подразумевается алгебраическое дополнение элемента матрицы a_{ij}. При помощи данного свойства можно вычислять не только матрицы третьего порядка, но и матрицы более высших порядков (4 x 4 или 5 x 5).Другими словами – это рекуррентная формула, которая нужна для того, чтобы вычислить определитель матрицы любого порядка. Запомните её, так как она часто применяется на практике.

Стоит сказать, что при помощи девятого свойства можно вычислять определители матриц не только четвёртого порядка, но и более высших порядков. Однако, при этом нужно совершать очень много вычислительных операций и быть внимательным, так как малейшая ошибка в знаках приведёт к неверному решению. Матрицы высших порядков удобнее всего решать методом Гаусса, и об этом поговорим позже.

10. Определитель произведения матриц одного порядка равен произведению их определителей.

Рассмотрим на примере:

Задача

Убедитесь, что определитель двух матриц A и B равен произведению их определителей. Даны две матрицы:

A = begin{vmatrix} 5&2\ -2&-4 end{vmatrix} right B = begin{vmatrix} 3&-1\ 1&6 end{vmatrix} right

Решение

Сначала находим произведение определителей двух матриц A и B.

|A| = begin{vmatrix}  5&2\  -2&-4  end{vmatrix} = 5 * (-4) - 2 * (-2) = (-20) - (-4) = -16  right

|B| = begin{vmatrix}  3&-1\  1&6  end{vmatrix} = 3 * 6 - (-1) * 1 = 18 - (1) = 19  right

|A| x |B| = -16 * 19 = -304,

Теперь выполним умножение обеих матриц и таким образом, вычислим определитель:

A * B = begin{pmatrix}  5&2\  -2&-4  end{pmatrix} * begin{pmatrix}  3&-1\  1&6  end{pmatrix}rightlongleftrightarrow

begin{pmatrix} 5 * 3 + 2 * 1&quadquad{5 * (- 1)+ 2 * 6}\(-2) * 3 + (-4) * 1&quadquad{(-2) * (-1) + (-4) * 6}  end{pmatrix} rightlongleftrightarrow begin{pmatrix} 15 + 2quadquad{(-5) + 12}\(-6) + (-4)quadquad{2 + (-24)end{pmatrix}rightlongleftrightarrow  begin{pmatrix}17&7\(-10)&(-22)end{pmatrix} =  17 * (-22) - 7 * (-10) = -374 + 70 = - 304 right

Ответ

Мы убедились, что |A * B| = |A| * |B|

Вычисление определителя матрицы при помощи метода Гаусса

Вспомним, как метод Гаусса помогает находить определитель матрицы: благодаря элементарным преобразованием в матрице все элементы (кроме a_{11}) нужно привести к нулю. Однако, такой метод подходит только к тем матрицам, в которых определитель отличен от нуля. Об этом поговорим позже, а сейчас объясним, для чего проделывается такая процедура.

Нулевые элементы необходимы для того, чтобы самым простым способом разложить определитель, исходя из элементов первого столбца. После такого преобразования, исходя из девятого свойства  и a_{21} = 0, a_{31} = 0,dots, a_{n_1} = 0, получается:

|A| = a_{11} * A_{11} + a_{21} * A_{21} + dots + a_{n1} * A_{n1} = a_{11} * A_{11} = a_{11} * (-1)^{1 + 1} * M_{11} = a_{11} * M_{11}.

Здесь M_{11} – это минор первого порядка, который получился из матрицы A путём вычёркивания элементов первой строки и первого столбца. Такая процедура проделывается до тех пор, пока все элементы первого столбца не превратятся в нулевые элементы.

Конечно же, сразу же назревает вопрос: “А как же получается нулевые элементы?” Рассмотрим алгоритм решения:

Если первый элемент в первой строке и в первом столбце (a_{11}) прибавить к соответствующим элементом k – ой строки, где a_{k1}neq{0}. (Метод Гаусса не нужен только в том случае, если все элементы в первом столбцы нулевые). После данного преобразования “новый” элемент матрицы a_{11}neq{0}. Определитель “новой” матрицы равен определителю исходной матрицы.

Если a_{11}neq{0}, тогда к каждому элементу второй строки прибавляем элемент первой строки, которые заранее умноженные на -{a_{21}over{a_{11}}}, а к элементам третьей строки прибавляем определённые элементы первой строки, которые умножаются на -{a_{31}over{a_{11}}}. И дальше вычисляем по такой же схеме. Метод Гаусса рассмотрен более подробно в отдельно теме. В итоге получится преобразованная матрица, где все элементы первого столбца окажутся нулевыми. Определитель полученной матрицы будет равен определителю изначальной матрицы.

Напомним, что величина определителя n – ого порядка равна сумме произведений элементов какой-либо строки или столбца на соответствующее алгебраическое дополнение.

Рассмотрим записанный сначала формально определитель четвёртого порядка:

Delta = begin{vmatrix} a_{11}&a_{12}&a_{13}&a_{14}\ a_{21}&a_{22}&a_{23}&a_{24}\ a_{31}&a_{32}&a_{33}&a_{34}\ a_{41}&a_{42}&a_43}&1a_{44} end{vmatrix} right

Вычёркивая в Delta i – тую строку и j – тый столбец, на пересечении которого помещается элемент a_{ij}(i, j = 1, 2, 3, 4), получим определитель третьего порядка, который называется минором элемента a_{ij} и обозначается M_{ij}. Тогда A_{ij} = (-1)^{i + j} * M – алгебраическое дополнение элемента a_{ij}. Определитель 4-го порядка можно обозначить, как размещение по элементам, например, первого столбца:

Delta = a_{11} * A_{11} + a_{21} * A_{21} + a_{31} * A_{31} + a_{41} * A_{41}.

Пусть введено понятие определителя (n - 1) – ого порядка, тогда определитель n – ого порядка:

Delta = begin{vmatrix} a_{11}&a_{12}&dots&a_{1j}&dots&a_{1n}\ a_{21}&a_{22}&dots&a_{2j}&dots&a_{2n}\ dots&dots&dots&dots\ a_{i1}&a_{i2}&dots&a_{ij}&dots&a_{in}\ dots&dots&dots&dots\ a_{n1}&a_{n2}&dots&a_{nj}&dots&a_{nn} end{vmatrix} right

Можно изобразить, как размещение по элементам первого столбца:

Delta = a_{11} * A_{11} + a_{21} * A_{21} + dots + a_{i1} * A_{i1} + dots + a_{n1} * A_{n1} = sum_{i = 1}^nlimits,

где A_{i1} = (-1)^{i + 1}quad{M{i1}(i = 1, 2, dots, n)} – алгебраические дополнения, а M{i1} – миноры элементов первого столбца. Последние и есть определители (n - 1) – го порядка.

Чтобы было более понятно, разберём матрицу четвёртого порядка, где нужно найти определитель:

Разберём на примере:

Задача

Нужно вычислить определитель матрицы высшего порядка 4 x 4:

Delta = begin{vmatrix} 1&-2&-1&3\ -1&3&-1&-1\ 3&-8&7&7\ 2&1&-10&17 end{vmatrix} right

Решение

Сначала вспомним тему про определители третьего порядка и превратим в нули элементы 1-го столбца, которые принадлежат 2, 3, 4 строкам. Для этого прибавим соответствующие элементы 1 и 2 строк. На месте элементов a_{21} получим (1 + (-1)) = 0, a_{22} = (-2) + 3 = 1, a_{23} = (-1) + (-1) = -2, a_{24} = 3 + (-1) = 2.

Чтобы получить 0 в 3 строке 1-го столбца, умножим на (-3) элементы 1-ой строки и прибавим к соответствующим элементам третьей строки:

a_{31} = 1 * (-3) + 3 = 0,

a_{32} = (-2) * (-3) + (-8) = - 2,

a_{33} = (-1) * (-3) + 7 = 10,

a_{34} = 3 * (-3) + 7 = -2.

Умножим элементы 1-ой строки на (-2) и добавим к соответствующим элементам 4-ой строки. Получается:

a_{41} = 1 * (-2) + 2 = 0,

a_{42} = (-2) * (-2) + 1 = 5,

a_{43} = (-1)*(-2) +(-10) = -8

a_{44} = 3 * (-2) + 17 = 11.

Изначальный определитель впоследствии преобразований получается:

Delta = begin{vmatrix} 1&-2&-1&3\ -1&3&-1&-1\ 3&-8&7&7\ 2&1&-10&17 end{vmatrix} = begin{vmatrix} 1&-2&-1&3\ 0&1&-2&2\ 0&-2&10&-2\ 0&5&-8&11 end{vmatrix} right

Дальше раскладываем последний определитель за элементами 1-го столбца. Поскольку a_{11} = 1, а остальные элементы 1-го столбца нули, тогда получим один определитель  3-го порядка.

Delta = begin{vmatrix} 1&-2&2\ -2&10&-2\ 5&-8&11 end{vmatrix} = begin{vmatrix} 1&-2&2\ 0&6&2\ 0&2&1 end{vmatrix} = begin{vmatrix} 6&2\ 2&1 end{vmatrix} = 2 right

Ответ

Определитель матрицы четвёртого порядка = 2.

Вычисление определителя матрицы при помощи теоремы Лапласа

Теорема Лапласа – это глубокое разложение определителя по элементам. При помощи данной теоремы можно решать матрицы не только третьего порядка, но и более высших порядков.

Напомним – минор – это определитель матрицы, который составлен методом вычёркивания i – той строки и j – того столбца. А алгебраическое дополнение – соответствующий минор, который берётся со знаком минус (-1)^{i + j}. Знаки же зависят от места элемента a_{ij} в определителе и определяются по схеме:

 begin{vmatrix} +&-&+\ -&+&-\ +&-&+ end{vmatrix} right

Приведём пример решения алгебраических дополнений по схеме:

Задача

Найти алгебраические дополнения элементов определителя:

 begin{vmatrix} 3&-2&4\ 0&-5&1\ 6&7&8 end{vmatrix} right

Решение

A_{11} = + begin{vmatrix}  -5&1\  7&8  end{vmatrix} = - 47; {A_{12}} = - begin{vmatrix}  0&1\  6&8  end{vmatrix} = 6; {A_{13}} = + begin{vmatrix}  0&-5\  6&7  end{vmatrix} = 30  right

A_{21} = - begin{vmatrix}  -2&4\  7&8  end{vmatrix} = 44; {A_{22}} = + begin{vmatrix}  3&4\  6&8  end{vmatrix} = 0; {A_{23}} = - begin{vmatrix}  3&-2\  6&7  end{vmatrix} = - 33  right

A_{31} = + begin{vmatrix}  -2&4\  -5&1  end{vmatrix} = 18; {A_{32}} = - begin{vmatrix}  3&4\  0&1  end{vmatrix} = - 3;{A_{33}} = + begin{vmatrix}  3&-2\  0&-5 end{vmatrix} = - 15 right

Понятия алгебраического дополнения даёт возможность ещё одного способа определения определителя, который утверждается теоремой Лапласа (про распределение определителя):

Определитель равняется сумме произведения элементов строк (столбца) на их алгебраические дополнения. Например,

Delta = a_{11} * A_{11} + a_{21} * A_{21} + a_{31} * a_{31}. – это равенство проверяется непосредственно

Delta = begin {vmatrix}  a_{11}&a_{12}&a_{13}\  a_{21}&a_{22}&a_{23}\  a_{31}&a_{32}&a_{33}  end{vmatrix} = a_{11} * A_{11} + a_{21} * A_{21} + a_{31} * A_{31} =\=a_{11}begin {vmatrix}a_{22}&a_{23\a_{32}&a_{33}end{vmatrix} - a_{21}begin {vmatrix}a_{12}&a_{13}\a_{32}&a_{33}end{vmatrix} + a_{31} begin {vmatrix}a_{12}&a_{13}\a_{22}&a_{23}end{vmatrix} = a_{11}a_{22}a_{33} - a_{11}a_{32}a_{23} -\- a_{21}a_{12}a_{33} + a_{21}a_{32}a_{13} + a_{31}a_{12}a_{23} - a_{31}a_{22}a_{13}.  right

Заметно, как последнее выражение совпадает с выражением из правила треугольника (правила Саррюса). Давайте по теореме Лапласа разберём несколько примеров:

Задача

Вычислить определитель матрицы, разложив его за элементами третьего порядка:

Delta = begin{vmatrix} 1&1&1\ 4&5&9\ 16&25&81 end{vmatrix} right

Решение

Delta = 16 * begin{vmatrix} 1&1\ 5&9 end{vmatrix} - 25 * begin{vmatrix} 1&1\ 4&9 end{vmatrix} + 81 * begin{vmatrix}1&1\ 4&5 end{vmatrix} = 16 * 4 - 25 * 5 + 81 * 1 = \ = 145 - 125 = 20 right

Ответ

Delta = 20.

Заключение

Итак, определитель квадратной матрицы – это число, полученное при помощи заданных чисел, расположенных в виде квадратной таблицы,которое вычисляется по рассмотренным выше формулам. Мы рассмотрели три основных способа вычисления определителя:

  1. через сумму двух произведений сочетаний элементов квадратной матрицы;
  2. по правилу разложения определителя по элементам строк (столбцов) квадратной матрицы;
  3. по методу Гаусса, когда матрицу нужно привести к треугольному виду.

Также были рассмотрены формулы для решения матрицы второго, третьего и высших порядков.

Мы разобрали 10 свойств определителя матриц, благодаря которым можно быстрее и легче найти определитель матрицы.

Удобно решать матрицу третьего порядка методом Гаусса, где нужно выполнить элементарные преобразования матрицы и привести её к ступенчатому виду. Определитель матрицы равняется произведению элементов, которые стоят на главной диагонали.

Полезная литература

pdf Белоусов И. В. Матрицы и определители, учеб. Пособие по линейной по алгебре/ – Кишинёв – 2006 – 91 с.

pdf Магазинников Л. И.- Практикум по линейной алгебре и аналитической геометрии: учеб. пособие для вузов/Магазинников Л. И., Магазинникова А. Л. – Томск – 2007 – 150 с.

Автор статьи

Эксперт по предмету «Математика»

Задать вопрос автору статьи

Любой квадратной матрице $A=left(a_{ij} right)_{ntimes n} $ можно сопоставить некоторое число, которое будем называть определителем данной матрицы (детерминант).

Для обозначения определителя матрицы используют следующие символы: $|A|,, Delta $ или $det A$.

В зависимости от порядка матрицы различают несколько способов вычисления определителя.

Определитель матрицы 2-го порядка можно вычислить по формуле:

Пример 1

Дана матрица $A=left(begin{array}{cc} {1} & {-2} \ {3} & {1} end{array}right)$. Найти определитель.

Решение:

[det A=left|begin{array}{cc} {1} & {-2} \ {3} & {1} end{array}right|=1cdot 1-3cdot (-2)=1+6=7]

Для нахождения определителя матрицы 3-го порядка можно использовать одно из двух правил:

  • правило треугольника;
  • правило Саррюса.

Определитель матрицы 3-го порядка с помощью правила треугольника вычисляется по формуле:

[left|begin{array}{ccc} {a_{11} } & {a_{12} } & {a_{13} } \ {a_{21} } & {a_{22} } & {a_{23} } \ {a_{31} } & {a_{32} } & {a_{33} } end{array}right|=]

[=a_{11} cdot a_{22} cdot a_{33} +a_{31} cdot a_{12} cdot a_{23} +a_{21} cdot a_{32} cdot a_{13} -a_{31} cdot a_{22} cdot a_{13} -a_{21} cdot a_{12} cdot a_{33} -a_{11} cdot a_{23} cdot a_{32} ]

Для лучшего запоминания правила треугольника можно пользоваться следующей схемой:

Для лучшего запоминания правила треугольника

Пример 2

Дана матрица $A=left(begin{array}{ccc} {1} & {3} & {4} \ {0} & {2} & {1} \ {1} & {5} & {-1} end{array}right)$. Найти определитель.

Решение:

[begin{array}{l} {det A=left|begin{array}{ccc} {1} & {3} & {4} \ {0} & {2} & {1} \ {1} & {5} & {-1} end{array}right|=1cdot 2cdot (-1)+1cdot 3cdot 1+4cdot 0cdot 5-1cdot 2cdot 4-0cdot 3cdot (-1)-5cdot 1cdot 1=} \ {=-2+3+0-8-0-5=-12} end{array}]

Для вычисления определителя по правилу Саррюса необходимо выполнить следующие действия:

  • дописать слева от определителя для первых столбца;
  • перемножить элементы, расположенные на главной диагонали и параллельных ей диагоналях, взяв произведения со знаком «+»;
  • перемножить элементы побочной диагонали и параллельных ей диагоналей, взяв произведения со знаком «-»

[left|begin{array}{ccc} {a_{11} } & {a_{12} } & {a_{13} } \ {a_{21} } & {a_{22} } & {a_{23} } \ {a_{31} } & {a_{32} } & {a_{33} } end{array}right|=]

[=a_{11} cdot a_{22} cdot a_{33} +a_{12} cdot a_{23} cdot a_{31} +a_{13} cdot a_{21} cdot a_{32} -a_{13} cdot a_{22} cdot a_{31} -a_{11} cdot a_{23} cdot a_{32} -a_{12} cdot a_{21} cdot a_{33} ]

Для лучшего запоминания правила треугольника можно пользоваться следующей схемой:

Для лучшего запоминания правила треугольника

«Определитель матрицы и его вычисление» 👇

Пример 3

Дана матрица $A=left(begin{array}{ccc} {1} & {3} & {4} \ {0} & {2} & {1} \ {-2} & {5} & {-1} end{array}right)$. Найти определитель.

Решение:

[begin{array}{l} {A=left|begin{array}{ccc} {1} & {3} & {4} \ {0} & {2} & {1} \ {-2} & {5} & {-1} end{array}right|, , , begin{array}{c} {1} \ {0} \ {-2} end{array}, , , , begin{array}{c} {3} \ {2} \ {5} end{array}=1cdot 2cdot (-1)+3cdot 1cdot (-2)+4cdot 0cdot 5-4cdot 2cdot (-2)-1cdot 1cdot 5-3cdot 0cdot (-1)=} \ {=-2-6+0+16-5-0=3} end{array}]

Для вычисления определителя матрицы 4-го порядка и выше можно использовать один из двух способов:

  • разложение по элементам строки;
  • разложение по элементам столбца.

Данные способы сводят вычисление определителя порядка $n$ к вычислению определителя порядка $n-1$ за счет представления определителя суммой произведений элементов строки (столбца) на их алгебраические дополнения.

Разложение определителя матрицы по элементам строки в общем виде можно записать по формуле:

[det A=a_{i1} cdot A_{i1} +a_{i2} cdot A_{i2} +…+a_{in} cdot A_{in} ]

Разложение определителя матрицы по элементам столбца в общем виде можно записать по формуле:

[det A=a_{1j} cdot A_{1j} +a_{2j} cdot A_{2j} +…+a_{nj} cdot A_{nj} ]

Замечание 1

При разложении определителя по элементам строки (столбца) желательно выбирать строку (столбец), в которой(-ом) есть нули.

Пример 3

Дана матрица $A=left(begin{array}{cccc} {0} & {1} & {-1} & {3} \ {2} & {1} & {0} & {0} \ {-2} & {4} & {5} & {1} \ {3} & {2} & {1} & {0} end{array}right)$. Записать разложение определителя по произвольной строке (столбцу).

Решение:

  • разложение по второй строке:
  • [A=left|begin{array}{cccc} {0} & {1} & {-1} & {3} \ {2} & {1} & {0} & {0} \ {-2} & {4} & {5} & {1} \ {3} & {2} & {1} & {0} end{array}right|=2cdot (-1)^{3} cdot left|begin{array}{ccc} {1} & {-1} & {3} \ {4} & {5} & {1} \ {2} & {1} & {0} end{array}right|+1cdot (-1)^{4} cdot left|begin{array}{ccc} {0} & {-1} & {3} \ {-2} & {5} & {1} \ {3} & {1} & {0} end{array}right|=-2cdot left|begin{array}{ccc} {1} & {-1} & {3} \ {4} & {5} & {1} \ {2} & {1} & {0} end{array}right|+1cdot left|begin{array}{ccc} {0} & {-1} & {3} \ {-2} & {5} & {1} \ {3} & {1} & {0} end{array}right|]

  • разложение по четвертому столбцу:
  • [A=left|begin{array}{cccc} {0} & {1} & {-1} & {3} \ {2} & {1} & {0} & {0} \ {-2} & {4} & {5} & {1} \ {3} & {2} & {1} & {0} end{array}right|=3cdot (-1)^{5} cdot left|begin{array}{ccc} {2} & {1} & {0} \ {-2} & {4} & {5} \ {3} & {2} & {1} end{array}right|+1cdot (-1)^{7} cdot left|begin{array}{ccc} {0} & {1} & {-1} \ {2} & {1} & {0} \ {3} & {2} & {1} end{array}right|=-3cdot left|begin{array}{ccc} {2} & {1} & {0} \ {-2} & {4} & {5} \ {3} & {2} & {1} end{array}right|-1cdot left|begin{array}{ccc} {0} & {1} & {-1} \ {2} & {1} & {0} \ {3} & {2} & {1} end{array}right|]

Свойства определителя:

  • элементарные преобразования строк (столбцов) матрицы не меняют значения определителя;
  • перестановка строк (столбцов) меняет знак определителя на противоположный (c «+» на «-» и наоборот);
  • определитель треугольной матрицы находится как произведение элементов, которые расположены на главной диагонали.

Пример 4

Дана матрица $A=left(begin{array}{ccc} {1} & {3} & {4} \ {0} & {2} & {1} \ {0} & {0} & {5} end{array}right)$. Найти определитель.

Решение:

[det A=left|begin{array}{ccc} {1} & {3} & {4} \ {0} & {2} & {1} \ {0} & {0} & {5} end{array}right|=1cdot 2cdot 5=10]

Замечание 2

Определитель матрицы, содержащей нулевую строку, равен нулю.

Замечание 3

Определитель матрицы, содержащей нулевой столбец, равен нулю.

Находи статьи и создавай свой список литературы по ГОСТу

Поиск по теме

Понравилась статья? Поделить с друзьями:
  • Как найти татьяну овсиенко
  • Как найти главу манги по фото
  • Как найти справку с места работы
  • Как найти друга в фифа мобайл 2022
  • Как найти перепутанную фазу